volume 25 • number 10 in this issue · 2011-07-12 · electrolyte abnormalities are common in...

25
Volume 25 • Number 10 June 2011 n Also in This Issue The LLSA Literature Review / Page 12 The Critical ECG / Page 13 The Critical Image / Page 20 CME Questions / Page 21 The Drug Box / Page 24 n Next Month The Hot Joint Fractures in Child Abuse In This Issue Lesson 19 Electrolyte Management in the Emergency Department Page 2 Electrolyte abnormalities are common in emergency medicine, and presentations can vary greatly, from asymptomatic patients to those with life-threatening arrhythmias and profound mental status changes. In these latter cases, emergency physicians must be prepared to diagnose and institute immediate treatment, often without awaiting laboratory results, to prevent cardiac arrest. Lesson 20 Cardiac Troponin Page 14 Elevated cardiac troponin concentrations have become a useful marker for acute coronary syndrome, but the definition of “elevated” is problematic, and other conditions, too, can cause increased troponin levels. Emergency physicians must understand the strengths and limitations of this marker in identifying acutely ill patients. Contributors Camiron L. Pfennig, MD, Sage Whitmore, MD, and Corey Slovis, MD, FACEP, wrote “Electrolyte Management in the Emergency Department. ” Dr. Pfennig is assistant professor of emergency medicine and director of undergraduate medical education in the Department of Emergency Medicine at Vanderbilt University in Nashville, Tennessee. Dr. Whitmore is a resident in the Department of Emergency Medicine at Vanderbilt University in Nashville. Dr. Slovis is professor of medicine and emergency medicine, chairman of the Department of Emergency Medicine at Vanderbilt University School of Medicine, and medical director of the Nashville Fire Department and International Airport in Nashville. George L. Sternbach, MD, FACEP, reviewed “Electrolyte Management in the Emergency Department.” Dr. Sternbach is a clinical professor of surgery at Stanford University Medical Center in Stanford, California, and an emergency physician at Seton Medical Center in Daly City, California. Alexander T. Limkakeng, Jr, MD, FACEP, and Jesmin Ehlers, MD, wrote “Cardiac Troponin.” Dr. Limkakeng is assistant professor and director of acute care in the Division of Emergency Medicine at Duke University Medical Center in Durham, North Carolina. Dr. Ehlers is a resident physician in the Division of Emergency Medicine at Duke University Medical Center in Durham. Amal Mattu, MD, FACEP, reviewed “Cardiac Troponin.” Dr. Mattu is program director of the Emergency Medicine Residency Program and professor of emergency medicine at the University of Maryland School of Medicine in Baltimore. Frank LoVecchio, DO, MPH, FACEP, reviewed the questions for these lessons. Dr. LoVecchio is research director at the Maricopa Medical Center Emergency Medicine Program and medical director of the Banner Poison Control Center, Phoenix, Arizona, and a professor at Midwestern University/Arizona College of Osteopathic Medicine in Glendale, Arizona. Louis G. Graff IV, MD, FACEP, is Editor-in-Chief of Critical Decisions. Dr. Graff is professor of traumatology and emergency medicine at the University of Connecticut School of Medicine in Farmington, Connecticut. Contributor Disclosures. In accordance with ACCME Standards and ACEP policy, contributors to Critical Decisions in Emergency Medicine must disclose the existence of significant financial interests in or relationships with manufacturers of commercial products that might have a direct interest in the subject matter. Individuals in control of content of these Critical Decisions lessons reported no such interests or relationships. Method of Participation. This educational activity consists of two lessons with a posttest, evaluation questions, and a pretest; it should take approximately 5 hours to complete. To complete this educational activity as designed, the participant should, in order, take the pretest (posted online following the previous month’s posttest), review the learning objectives, read the lessons as published in the print or online version, and then complete the online posttest and evaluation questions. Release date June 1, 2011. Expiration date May 31, 2014. Accreditation Statement. The American College of Emergency Physicians is accredited by the ACCME to provide continuing medical education for physicians. The American College of Emergency Physicians designates this enduring material for a maximum of 5 AMA PRA Category 1 Credits™. Physicians should claim only the credit commensurate with the extent of their participation in the activity. Each issue of Critical Decisions in Emergency Medicine is approved by ACEP for 5 ACEP Category I credits. Approved by the AOA for 5 Category 2-B credits (a minimum score of 70% is required). Commercial Support. There was no commercial support for this CME activity. Target Audience. This educational activity has been developed for emergency physicians.

Upload: others

Post on 31-May-2020

3 views

Category:

Documents


0 download

TRANSCRIPT

Page 1: Volume 25 • Number 10 In This Issue · 2011-07-12 · Electrolyte abnormalities are common in emergency medicine, but the degree of severity can vary greatly. Asymptomatic electrolyte

Volume 25 • Number 10

June

2011

n Also in This Issue• TheLLSALiteratureReview

/Page12

• TheCriticalECG/Page13

• TheCriticalImage/Page20

• CMEQuestions/Page21

• TheDrugBox/Page24

n Next Month• TheHotJoint

• FracturesinChildAbuse

In This IssueLesson 19 Electrolyte Management in the Emergency Department . . . . . Page 2

Electrolyte abnormalities are common in emergency medicine, and presentations can vary greatly, from asymptomatic patients to those with life-threatening arrhythmias and profound mental status changes. In these latter cases, emergency physicians must be prepared to diagnose and institute immediate treatment, often without awaiting laboratory results, to prevent cardiac arrest.

Lesson 20 Cardiac Troponin . . . . . . . . . . . . . . . . . . . . . . . . . . . . . . . . . . . . . Page 14Elevated cardiac troponin concentrations have become a useful marker for acute coronary syndrome, but the definition of “elevated” is problematic, and other conditions, too, can cause increased troponin levels. Emergency physicians must understand the strengths and limitations of this marker in identifying acutely ill patients.

ContributorsCamiron L. Pfennig, MD, Sage Whitmore, MD,andCorey Slovis, MD, FACEP,wrote“Electrolyte

Management in the Emergency Department.”Dr.PfennigisassistantprofessorofemergencymedicineanddirectorofundergraduatemedicaleducationintheDepartmentofEmergencyMedicineatVanderbiltUniversityinNashville,Tennessee.Dr.WhitmoreisaresidentintheDepartmentofEmergencyMedicineatVanderbiltUniversityinNashville.Dr.Slovisisprofessorofmedicineandemergencymedicine,chairmanoftheDepartmentofEmergencyMedicineatVanderbiltUniversitySchoolofMedicine,andmedicaldirectoroftheNashvilleFireDepartmentandInternationalAirportinNashville.

George L. Sternbach, MD, FACEP,reviewed“ElectrolyteManagementintheEmergencyDepartment.”Dr.SternbachisaclinicalprofessorofsurgeryatStanfordUniversityMedicalCenterinStanford,California,andanemergencyphysicianatSetonMedicalCenterinDalyCity,California.

Alexander T. Limkakeng, Jr, MD, FACEP,andJesmin Ehlers, MD,wrote“Cardiac Troponin.”Dr.LimkakengisassistantprofessoranddirectorofacutecareintheDivisionofEmergencyMedicineatDukeUniversityMedicalCenterinDurham,NorthCarolina.Dr.EhlersisaresidentphysicianintheDivisionofEmergencyMedicineatDukeUniversityMedicalCenterinDurham.

Amal Mattu, MD, FACEP,reviewed“CardiacTroponin.”Dr.MattuisprogramdirectoroftheEmergencyMedicineResidencyProgramandprofessorofemergencymedicineattheUniversityofMarylandSchoolofMedicineinBaltimore.

Frank LoVecchio, DO, MPH, FACEP,reviewedthequestionsfortheselessons.Dr.LoVecchioisresearchdirectorattheMaricopaMedicalCenterEmergencyMedicineProgramandmedicaldirectoroftheBannerPoisonControlCenter,Phoenix,Arizona,andaprofessoratMidwesternUniversity/ArizonaCollegeofOsteopathicMedicineinGlendale,Arizona.

Louis G. Graff IV, MD, FACEP,isEditor-in-ChiefofCritical Decisions.Dr.GraffisprofessoroftraumatologyandemergencymedicineattheUniversityofConnecticutSchoolofMedicineinFarmington,Connecticut.Contributor Disclosures. InaccordancewithACCMEStandardsandACEPpolicy,contributorstoCritical Decisions in Emergency Medicinemustdisclosetheexistenceofsignificantfinancialinterestsinorrelationshipswithmanufacturersofcommercialproductsthatmighthaveadirectinterestinthesubjectmatter.IndividualsincontrolofcontentoftheseCritical Decisionslessonsreportednosuchinterestsorrelationships.Method of Participation. Thiseducationalactivityconsistsoftwolessonswithaposttest,evaluationquestions,andapretest;itshouldtakeapproximately5hourstocomplete.Tocompletethiseducationalactivityasdesigned,theparticipantshould,inorder,takethepretest(postedonlinefollowingthepreviousmonth’sposttest),reviewthelearningobjectives,readthelessonsaspublishedintheprintoronlineversion,andthencompletetheonlineposttestandevaluationquestions.ReleasedateJune1,2011.ExpirationdateMay31,2014.Accreditation Statement.TheAmericanCollegeofEmergencyPhysiciansisaccreditedbytheACCMEtoprovidecontinuingmedicaleducationforphysicians.TheAmericanCollegeofEmergencyPhysiciansdesignatesthisenduringmaterialforamaximumof5AMA PRA Category 1 Credits™.Physiciansshouldclaimonlythecreditcommensuratewiththeextentoftheirparticipationintheactivity.EachissueofCritical Decisions in Emergency MedicineisapprovedbyACEPfor5ACEPCategoryIcredits.ApprovedbytheAOAfor5Category2-Bcredits(aminimumscoreof70%isrequired).Commercial Support.TherewasnocommercialsupportforthisCMEactivity.Target Audience. Thiseducationalactivityhasbeendevelopedforemergencyphysicians.

Page 2: Volume 25 • Number 10 In This Issue · 2011-07-12 · Electrolyte abnormalities are common in emergency medicine, but the degree of severity can vary greatly. Asymptomatic electrolyte
Page 3: Volume 25 • Number 10 In This Issue · 2011-07-12 · Electrolyte abnormalities are common in emergency medicine, but the degree of severity can vary greatly. Asymptomatic electrolyte

CriticalDecisionsinEmergencyMedicine

2

CamironL.Pfennig,MD,SageWhitmore,MD,andCoreySlovis,MD,FACEP

n ObjectivesOn completion of this lesson, you should be able to:

1. Listcommondiseasestatesthatplacepatientsatriskforseriouselectrolytedisturbances.

2. Describetheclassicsignsandsymptomsofmajorelectrolytedisturbances.

3. Statewhichsymptomsorobjectivefindingsofsevereelectrolytedisturbanceswarrantemergentcorrection.

4. Describetheemergencytreatmentsforeachelectrolyteabnormality.

5. DiscusstheECGchangesthatarecharacteristicofhyperkalemiaandhypokalemia.

6. Explainwhichelectrolytedisturbancesleadtomalignantarrhythmias.

7. Describewhichpatientsrequireadmissionformanagementoftheirelectrolytedisturbances.

n From the EM Model5.0 Endocrine,Metabolic,and

NutritionalDisorders

5.3 FluidandElectrolyteDisturbances

Electrolyte Management in the Emergency Department

Lesson 19

Electrolyte abnormalities are common in emergency medicine, but the degree of severity can vary greatly. Asymptomatic electrolyte abnormalities can usually be slowly corrected, but those that cause profound mental status changes or life-threatening arrhythmias require immediate correction to avoid cardiac arrest or seizures. In some cases, life-threatening electrolyte abnormalities must be treated even before laboratory results become available.

Case Presentations

n Case OneA 51-year-old woman with

a history of diabetes mellitus, hypertension, peripheral vascular disease, and hemodialysis-dependent end-stage renal disease presents to the emergency department by ambulance because of dizziness, weakness, abdominal pain, vomiting, and diarrhea. Her symptoms began 3 days ago and have been worsening. She did not feel well enough to go to her dialysis appointment yesterday and has not been able to afford any of her medications this week.

On physical examination, the patient appears lethargic and very ill. Her initial vital signs are supine blood pressure 98/66, pulse rate 98, respiratory rate 26, oral temperature 36.7°C (98°F), and oxygen saturation 96% on room air. She has a patent airway and an unremarkable HEENT examination. Her chest examination reveals mild bibasilar crackles and normal heart sounds with II/VI systolic murmur. Her abdomen is

soft but diffusely tender. She has a weak radial pulse in the right arm; there is an AV fistula with a palpable thrill medial to the left biceps. Her lower extremities are cool, dry, and shiny, with 1+ pitting pretibial edema bilaterally.

By department protocol, in light of the patient’s initial vital signs and lethargy, weakness, and ill appearance, she was placed on a cardiac monitor and an ECG was performed immediately. The ECG and the rhythm on the monitor reveal a junctional tachycardia without P waves, a wide QRS, and peaked T waves. Blood is obtained by peripheral butterfly stick, and a stat chemistry panel reveals sodium 129, potassium 8.2, chloride 88, bicarbonate 5, BUN 48, creatinine 3.9, glucose 422, and venous pH 7.11.

n Case TwoA 22-year-old man is transported

to the emergency department from his fraternity house with new-onset seizure activity. The paramedics state that his fraternity brothers called the ambulance because of the patient’s confusion and vomiting that had been worsening over the past 6 hours. On EMS arrival, the patient began seizing, and lorazepam was administered with no improvement. His friends reported that he had no known medical problems and took no daily medications. Multiple empty beer cans were noted to be scattered around the house, but no empty pill bottles were seen.

Page 4: Volume 25 • Number 10 In This Issue · 2011-07-12 · Electrolyte abnormalities are common in emergency medicine, but the degree of severity can vary greatly. Asymptomatic electrolyte

June2011•Volume25•Number10

3

The patient’s vital signs on arriving in the emergency department are blood pressure 102/33, pulse rate 130, respiratory rate 12, oral temperature 36.7°C (98.1°F), and oxygen saturation 89% on 100% oxygen via nonrebreathing mask. His bedside glucose check is 178. The patient is intubated with no difficulties, and benzodiazepine administration is continued. Just as the physician is preparing a levetiracetam infusion, the patient’s friend arrives and reports that the patient had been drinking alcohol and using ecstasy at a party. Blood is obtained, and a chemistry panel reveals sodium 105, potassium 2.8, chloride 110, bicarbonate 18, BUN 50, creatinine 5, and glucose 169.

HyperkalemiaHyperkalemia, defined as a

serum potassium level of more than 5 mEq/L, is the most common electrolyte abnormality leading to life-threatening arrhythmias and

cardiac arrest.1 Hyperkalemia has vague and varied symptoms; in fact, it can be totally asymptomatic, or the initial presentation may be sudden death. The correct and early diagnosis of hyperkalemia requires attention to risk factors, especially a history of renal failure and medication use that causes potassium retention, as well as a search for ECG changes consistent with elevated potassium. Hyperkalemia can be rapidly progressive, and lifesaving interventions must be instituted at the earliest suspicion of toxicity. Table 1 organizes five of the most common causes of hyperkalemia. A history of one of these conditions may be the lone clue to the diagnosis because symptoms do not reliably appear with any particular serum potassium level.1

The ECG can be helpful in making the diagnosis of hyperkalemia. Peaked T waves appear as serum potassium levels exceed 5.5 to 6.5 mEq/L; P-wave disappearance and PR prolongation occur with levels

above 6.5 to 7.5 mEq/L; and QRS prolongation occurs when levels rise above 7 to 8 mEq/L (Figure 1).1-3 These ECG changes occur in only half of patients with hyperkalemia, but recognizing these changes, when they are present, is vital to rapid diagnosis and initiation of lifesaving treatment.4

A serum potassium level above 5 mEq/L is diagnostic of hyperkalemia, but the value itself does not always predict ECG changes or the degree of cardiotoxicity.5

Patients with suspected or known hyperkalemia should have an intravenous line established and should be placed on continuous cardiac monitoring. The treatment of hyperkalemia is based on the clinical scenario combined with the 12-lead ECG and the laboratory potassium value. The treatment strategy consists of three main steps: 1) stabilizing the cardiac membrane, 2) shifting potassium into the cells, and 3) removing potassium from the body.

CRITICAL DECISIONWhen should intravenous calcium be administered in patients with hyperkalemia?

Administration of calcium as either calcium chloride or calcium gluconate for hyperkalemia is controversial.1 Some authors advocate administering calcium for any ECG changes associated with hyperkalemia, including isolated peaked T waves. The authors believe that there are only three indications for administering calcium in hyperkalemia, as follows:

• Whenshouldintravenouscalciumbeadministeredinpatientswithhyperkalemia?

• Whatmedicationsshouldbeconsideredforpatientswithhyperkalemiatodrivepotassiumintothecell,aftertheimmediateneedforintravenouscalciumhasbeendetermined?

• Whenshouldhemodialysisbeinitiatedinahyperkalemicpatient?

• Howshouldapatientwithsuspectedhypokalemiabeevaluatedandmanaged?

• Whenshouldhypertonicsalinebeadministeredtohyponatremicpatients?Whatrisksmustemergencyphysiciansbeawareofwhentreatinghyponatremia?

• Howarethefourgeneralcategoriesofhyponatremiaidentified,andhowshouldeachbemanaged?

• Whenshouldanemergencyphysicianconsiderhypercalcemiainthedifferentialdiagnosis?Whattreatmentshouldbeinitiatedinhypercalcemiccrises?

• Whatimportantsignsandsymptomsshouldraiseconcernforhypocalcemia?Whattreatmentshouldbeconsideredinsymptomatichypocalcemia?

Critical Decisions

Table 1.Fivemostcommoncausesofhyperkalemia

Spurious elevation Hemolysis while drawing or storing the laboratory sample

Renal failure Acute or chronic

Acidosis Diabetic ketoacidosis, Addison disease, adrenal insufficiency, type 4 renal tubular acidosis

Cell death Rhabdomyolysis, tumor lysis syndrome, burns, massive hemolysis or transfusion, crush injury

Drugs -Blockers, acute digitalis toxicity, succinylcholine, ACE inhibitors, angiotensin receptor blockers, nonsteroidal anti-inflammatory drugs, spironolactone, amiloride

Page 5: Volume 25 • Number 10 In This Issue · 2011-07-12 · Electrolyte abnormalities are common in emergency medicine, but the degree of severity can vary greatly. Asymptomatic electrolyte

CriticalDecisionsinEmergencyMedicine

4

1) a widening QRS including a sine wave, 2) a cardiac arrest that is believed to be due to hyperkalemia, or 3) signs of rapidly progressing hyperkalemia in the face of tumor lysis syndrome, massive hemolysis, or rhabdomyolysis where a normal ECG has rapidly progressed through tall peaked T waves and loss of the P wave. In this situation, calcium could be given “prophylactically” while other methods are used to stop potassium release, drive already released potassium into the cell, and begin emergent dialysis to remove potassium from the body.

Because intravenous calcium can cause tachycardia, hypertension, and arrhythmias, as well as hypercalcemia, we urge that calcium not be given routinely for hyperkalemia in an otherwise stable patient with a normal QRS or just isolated, peaked T waves. In the context of digitalis toxicity, intravenous calcium should still be used for life-threatening hyperkalemia with a widening QRS while awaiting the administration and effects of digoxin Fab fragments.

Calcium restores the electrical and chemical gradient of the cardiac myocyte, thus narrowing the QRS.5 Calcium does not decrease serum potassium levels, and its effect is rapid but transient. The dose is one ampule, or 10 mL of 10% calcium chloride solution, with a maximum dose of two ampules or 20 mL. Some authors prefer calcium gluconate to calcium chloride based on the reduced risk of tissue necrosis should it extravasate at the injection site.6 Calcium gluconate may also be preferred in pediatric cases and in more chronic, less emergent hyperkalemic patients when a slow infusion is desired. It has about one third the amount of free calcium (13.6 mEq/10 mL for calcium chloride versus 4.6 mEq/10 mL for calcium gluconate).

CRITICAL DECISIONWhat medications should be considered for patients with hyperkalemia to drive potassium into the cell, after the immediate need for intravenous calcium has been determined?

A 2-agonist, insulin and glucose, in some cases sodium bicarbonate, and saline can be given to shift potassium into cells. Sodium bicarbonate alone does not lower serum potassium in patients with hyperkalemia and is unreliable at best in combination with other agents. It should be reserved for patients who are severely acidemic.3,7,8

Nebulized albuterol by face mask begins to take measurable effect after 15 to 20 minutes and lowers the serum potassium level by up to

1 mEq/L, depending on the dose. -Agonists are safe despite the side effect of tachycardia.9,10 Insulin, given intravenously in combination with glucose, also results in a similar fall in the potassium level after 20 to 30 minutes and also lowers levels by up to 1 mEq/L. The combination of nebulized albuterol and intravenous insulin with glucose appears to be additive, lowering serum potassium by a mean of 1.21 mEq/L or more.11 Adult hyperkalemic patients who have ECG changes should receive continuous nebulized albuterol and 50 grams of intravenous dextrose plus 10 units of intravenous regular insulin.

Most patients with hyperkalemia have impaired or no renal function. However, even a few hundred

Figure 1.ECGchangesinhyperkalemia

Tall, peaked T wave

Loss of P wave

Tall, peaked T wave Widened QRS mergingwith tall T wave

Sine wave

Figure 2.ECGchangesinhypokalemia

ST depression

Flat T waveU wave

Prolonged QT

Page 6: Volume 25 • Number 10 In This Issue · 2011-07-12 · Electrolyte abnormalities are common in emergency medicine, but the degree of severity can vary greatly. Asymptomatic electrolyte

June2011•Volume25•Number10

5

milliliters of normal saline can help move potassium intracellularly via the sodium-potassium pump. Prior to the use of saline, the patient’s nephrologist should be consulted and emergent hemodialysis scheduled. For patients with normal or near normal renal function such as those with rhabdomyolysis or tumor lysis syndrome, aggressive saline diuresis supplemented by furosemide may be all that is required to treat the patient’s hyperkalemia, thus avoiding dialysis.

Cation exchange resins, such as sodium polystyrene sulphonate have not been shown to decrease the serum potassium level within the first 4 hours of treatment and should not be used alone in the acute management of hyperkalemia.12 Table 2 outlines recommended treatment for hyperkalemia.

CRITICAL DECISIONWhen should hemodialysis be initiated in a hyperkalemic patient?

Emergent hemodialysis is the most reliable method of definitively lowering serum potassium in patients with renal failure. Hemodialysis reliably decreases serum potassium levels by at least 1 mEq/L in the first hour and another 1 mEq/L over the next 2 hours.7,9,10 It should be instituted early on in the treatment

of life-threatening hyperkalemia in patients with renal failure.10 Hemodialysis should also be the treatment of choice for hyperkalemic patients who have impaired renal function and pulmonary edema caused by fluid overload.

Hemodialysis via central venous access can be used during ongoing cardiopulmonary resuscitation to acutely lower the serum potassium level and may result in return of spontaneous circulation with intact neurologic status despite prolonged resuscitative efforts and failure of conventional medications and defibrillation.13

In patients with intact renal function, medical management alone may be sufficient even in extreme cases, and hemodialysis may not be necessary unless multiple medical modalities fail.8 These patients should initially be treated medically and hemodialysis delayed until it appears that medical management alone has failed.

HypokalemiaHypokalemia, defined as a

serum potassium level less than 3.5 mEq/L, is a very common electrolyte abnormality that is often asymptomatic.14 Hypokalemia is usually associated with hypomagnesemia.15 Although usually

asymptomatic, cardiac arrhythmias and rhabdomyolysis can occur from hypokalemic effects on the heart and muscle. The five most common causes of hypokalemia are listed in Table 3.

CRITICAL DECISIONHow should a patient with suspected hypokalemia be evaluated and managed?

Patients in whom severe hypokalemia is suspected should be placed on a cardiac monitor and intravenous access should be established immediately. An ECG should be obtained early in the patient’s evaluation. Just as a tall, peaked T wave is characteristic of hyperkalemia, a flattened T wave can be seen in hypokalemia. A U wave, a small deflection after the T wave, may also be seen.2 Hypokalemia is also notorious for causing nonspecific ST and T-wave changes (Figure 2). The real danger of hypokalemia is that it can also cause a prolonged QT interval.16 Once the QT interval becomes greater than 500 msec, the risk of malignant ventricular arrhythmias and torsade de pointes increases dramatically.17

A low serum potassium level reflects a much greater total-body potassium deficit; when treating hypokalemia in the emergency department, remember that a 0.3

Table 2.Stepsinthetreatmentofhyperkalemia

Treatment Medication Features

1. Stabilize cardiac membrane Calcium chloride or calcium gluconate, 1 to 2 ampules, IV push

For widening QRS; restores the electrical gradient, does not decrease serum potassium

2. Shift potassium into cells Nebulized albuterol by face maskInsulin, 10 units, IV push, combined with 50% dextrose, 2 ampules, IV push

Normal saline

Sodium bicarbonate Only if acidotic

3. Remove potassium from the body Hemodialysis Emergently in cardiac arrest, urgently in renal failure, may delay if normal renal function

Normal saline and furosemide In patients with rhabdomyolysis or tumor lysis syndrome with intact urine output

Cation exchange resins

Page 7: Volume 25 • Number 10 In This Issue · 2011-07-12 · Electrolyte abnormalities are common in emergency medicine, but the degree of severity can vary greatly. Asymptomatic electrolyte

CriticalDecisionsinEmergencyMedicine

6

mEq potassium drop below normal serum levels correlates with an approximately 100 mEq total-body deficit.18 Patients who have mild or moderate hypokalemia may need only oral potassium replacement therapy if nausea or vomiting is not the cause of the hypokalemia.19 Potassium chloride is the most commonly used supplement, and 40 to 60 mEq orally every 2 to 4 hours is typically well tolerated.20

If intravenous infusion is necessary, potassium chloride at a rate of 10 to 20 mEq/hr is considered very safe.21 Infused potassium can cause burning pain in patients with small veins, so small amounts of lidocaine may be added to the intravenous solution.22 When the intravenous repletion rate is faster than 20 mEq/hr, continuous cardiac monitoring is required and central line access is recommended. Constant bedside vigilance with close monitoring of the patient and ECG is required when administering more than 20 mEq/hr of potassium. Unless the physician gives at least 0.5 grams/hr of magnesium sulfate along with potassium replacement, the potassium cannot move intracellularly and the patient will lose essentially all the infused potassium as urinary losses.23 It is important to realize that correction of large potassium deficits may require several days and that oral and intravenous replacement can occur simultaneously.

HyponatremiaHyponatremia is defined as a

serum sodium concentration of less than 135 mEq/L and is the most common electrolyte abnormality encountered in clinical practice.24 Most patients presenting to the emergency department with hyponatremia are stable and require no emergency therapy. However, there are patients who will need treatment acutely in the emergency department.

CRITICAL DECISIONWhen should hypertonic saline be administered to hyponatremic patients? What risks must emergency physicians be aware of when treating hyponatremia?

Hypertonic saline (3% saline solution) can rapidly and effectively raise serum sodium levels but can also cause the disastrous complication of central pontine myelinolysis. Hypertonic saline should be reserved for severely hyponatremic patients with neurologic signs, including coma, seizures, and focal neurologic deficits, who have a serum sodium level below 120 mEq/L (usually in the 100 to 110 mEq/L range). Hypertonic saline is typically reserved for patients with an acute disease process rather than chronic hyponatremia.

Typically, sodium should be corrected over 48 to 72 hours. Central pontine myelinolysis has been reported when patients have had their serum sodium raised by more than 0.5 mEq/hr or 10 to 12 mEq/day. Thus, slow correction should always be the focus of therapy for hyponatremia. Hypertonic saline can rapidly increase serum sodium by 2 to 3 mEq/hr or more depending on the amount infused.

When rapid correction of the serum sodium concentration is needed, 3% hypertonic solutions should be administered at a rate of approximately 1 to 2 mL/kg/hr.25 Our recommendation is to give critically ill hyponatremic patients (those seizing, with focal findings, or with coma) 100 mL of 3% hypertonic saline over 10 minutes. If a second

bolus is required, give a second 100 mL of the 3% solution over the next 50 minutes. Potassium should also be replaced aggressively when treating hypokalemic patients with a sodium disorder. Physicians should supplement therapy with furosemide if the patient is retaining volume and not diuresing adequately.

Central pontine myelinolysis, a demyelinating disease of the pons and central nervous system, can occur with overly rapid correction of sodium.26 If fluid therapy raises extracellular sodium levels too quickly, fluids shift out of neurons and demyelinization may occur, leading to flaccid paralysis and often death.27 Patients with central pontine myelinolysis can present with flaccid paralysis, dysarthria, dysphagia, and hypotension. If a patient develops these symptoms during therapy, all sodium-containing fluids should be stopped and D5W immediately administered to again lower sodium values temporarily.

CRITICAL DECISIONHow are the four general categories of hyponatremia identified, and how should each be managed?

Although hyponatremia has many causes, they fall into four general categories, as follows: pseudohyponatremia, hyponatremia with dehydration and decreased extracellular volume, hyponatremia with increased extracellular volume, and euvolemic hyponatremia with increased total body water.

Pseudohyponatremia is a falsely low sodium reading

Table 3.Causesofhypokalemia

Renal losses Diuretic use, hyperaldosteronism, steroid excesses, metabolic acidosis, drugs, diabetic ketoacidosis, renal tubular acidosis, alcohol consumption

Increased nonrenal losses Sweating, diarrhea, vomiting, laxative use

Decreased intake Alcohol use, malnutrition

Intracellular shift Hyperventilation, metabolic alkalosis, drugs

Endocrine Cushing disease, Barter syndrome, insulin therapy

Page 8: Volume 25 • Number 10 In This Issue · 2011-07-12 · Electrolyte abnormalities are common in emergency medicine, but the degree of severity can vary greatly. Asymptomatic electrolyte

June2011•Volume25•Number10

7

caused by the presence of other osmolar particles in the serum such as hypertriglyceridemia and hyperproteinemia. Hyperglycemia can cause decreased sodium levels by pulling water into the vascular space by osmosis. In order to calculate the corrected sodium level, 1.6 mEq/L should be added to the measured sodium for every 100 mg/dL of glucose above 100 up to about 400, then 4 mEq/L should be added for every additional 100 mg/dL.28 We find it easier to just remember that the maximum fall for every 100-mg elevation in blood glucose is about 2.5 mEq/L of sodium. Emergency physicians should always consider that laboratory errors, or blood draw errors, may have caused the apparent hyponatremia, especially if the blood sample was drawn near an infusion site using D5W or D5½NS.

Hyponatremia with dehydration, also known as hypovolemic hyponatremia, occurs when there is decreased extracellular volume with an even greater loss of sodium. It is important to determine whether patients are hyponatremic because of body fluid losses or because of renal losses. Body fluid losses include sweating, vomiting, diarrhea, and third spacing, while renal losses include diuretic use, mineralocorticoid deficiency, renal tubular acidosis, and salt-wasting nephropathy. To determine if the

kidney is causing the hyponatremia, the physician should order a spot urinary sodium and/or urinary chloride. Patients with hypovolemic hyponatremia from nonrenal causes have renal absorption of tubular sodium and urine sodium levels of less than 20 mEq/L, whereas those with hypovolemic hyponatremia from renal causes have inappropriately elevated urine sodium levels in excess of 20 mEq/L. Treatment of hypovolemic hyponatremia begins with rehydration with normal saline. Hypotensive, dehydrated patients should be volume-resuscitated with normal saline, but once they are hemodynamically stable, the infusion rate should be slowed down.

In contrast, hyponatremia with increased extracellular volume occurs when sodium and water are retained but water retention exceeds sodium retention. Most of these patients present with edema. Hyponatremia with increased total body sodium occurs in patients with heart failure, chronic renal failure, and hepatic failure secondary to hypoperfusion of the kidneys causing high aldosterone secretion. Emergency physicians must be aware that normal saline and hypertonic saline can cause pulmonary edema in these patients. Patients with congestive heart failure contributing to their hyponatremia will usually benefit from diuretics to increase water excretion, along

with vasodilation to improve cardiac output.29 Dialysis may be required in renal failure patients. In patients with liver failure, the physician should consider albumin, diuretics, and possibly paracentesis to improve the underlying pathology. It is also very important to discuss water restriction with the admitting physician when patients are transferred to the inpatient service.

The final category of hyponatremia involves patients with euvolemia but increased total body water. Causes of this type of hyponatremia include syndrome of inappropriate antidiuretic hormone (SIADH), psychogenic polydipsia, beer potomania, hypothyroidism, diuretic use in patients with mild congestive heart failure, and accidental or intentional water intoxication.30 These patients do not present with edema because most of the increased body water is intracellular and not intravascular. The mainstay of treatment for euvolemic hyponatremia is free water restriction.31

HypercalcemiaThere are five major causes of

hypercalcemia (Table 4). Primary hyperparathyroidism is the most common cause of hypercalcemia in outpatients, while malignancy is the most common cause in hospitalized patients.32 Mild hypercalcemia in an otherwise healthy person could be caused by thiazide diuretics with minimal dehydration. Other causes of elevated calcium are less common and usually are not considered until malignancy and parathyroid disease are ruled out.

CRITICAL DECISIONWhen should an emergency physician consider hypercalcemia in the differential diagnosis? What treatment should be initiated in hypercalcemic crises?

Unfortunately, the clinical presentation of hypercalcemia is often vague and nonspecific, and includes nonfocal abdominal pain, constipation, fatigue, diffuse body aches, anorexia, nausea, and

Table 4.Causesofhypercalcemia

Malignant disease Ectopic secretions of parathyroid hormone, multiple myeloma, cancer metastatic to boneMost common: breast, lung, hematologic, kidney, prostate

Endocrine Hyperparathyroidism, multiple endocrine neoplasias, hyperthyroidism, pheochromocytoma, adrenal insufficiency

Granulomatous disease Sarcoidosis, tuberculosis, histoplasmosis, berylliosis, coccidioidomycosis

Pharmacological agents Vitamins A and D, thiazide diuretics, estrogens, milk-alkali syndrome

Miscellaneous Dehydration, prolonged immobilization, iatrogenic, rhabdomyolysis, familial, laboratory error

Page 9: Volume 25 • Number 10 In This Issue · 2011-07-12 · Electrolyte abnormalities are common in emergency medicine, but the degree of severity can vary greatly. Asymptomatic electrolyte

CriticalDecisionsinEmergencyMedicine

8

vomiting. The condition must be considered in a large number of differential diagnoses. In addition, some patients complain of polyuria or polydipsia. Hypercalcemia can also present in the emergency department with lethargy, altered mental status, coma, and seizures. The mnemonic “bones, moans, stones, and groans” provides a clue that hypercalcemia should be considered in the differential diagnosis (Table 5). The most common causes of death in hypercalcemia are complications caused by coma, dehydration, electrolyte disturbances, and cardiac abnormalities.

When trying to make the diagnosis of hypercalcemia, the total serum calcium is the only laboratory value required. Ionized calcium is the active form of the total calcium level and is much more accurate for correcting and diagnosing hypocalcemia.33 It is important to obtain an ECG in these patients as well. ECG changes in hypercalcemia include shortening of the QT interval and, rarely, QRS widening. Although a short QT interval is the “classic” finding in hypercalcemia, it is an unreliable finding, seen in only about 20% of patients. In severe cases, sinus bradycardia, bundle-branch block, and high-degree atrioventricular block can also be seen.34

Hypercalcemic patients should first have their “ABCs” secured and have intravenous access established. Patients in hypercalcemic crisis are usually dehydrated, obtunded, and predisposed to arrhythmias due to concomitant electrolyte disturbances. Once the primary survey has been completed, normal saline infusion should be initiated. Saline will inhibit proximal tubule resorption of calcium. The routine use of furosemide in the management of hypercalcemia is no longer recommended. Furosemide was once thought to block the distal reabsorption of calcium thus complementing saline’s proximal effects. No recent study has shown furosemide to have significant

calcium reabsorption-blocking effects, and its use should now be reserved to augment the saline-induced diuresis and to avoid volume overload.35 Definitive calcium-lowering therapy with bisphosphonates that specifically inhibit osteoclastic calcium production should be considered in consultation with the patient’s primary care physician, endocrinologist, oncologist, or other appropriate specialist.

HypocalcemiaSevere, symptomatic hypocalcemia

can result in cardiovascular collapse, hypotension, and arrhythmias.36,37 There are multiple causes of hypocalcemia with hypoalbuminemia being the most common cause. Although such entities as vitamin D deficiency, parathyroid disease, and surgery are often cited as common causes of hypocalcemia, Table 6 contains the five most common causes of acute, symptomatic hypocalcemia seen in emergency departments.

CRITICAL DECISIONWhat important signs and symptoms should raise concern for hypocalcemia? What treatment should be considered in symptomatic hypocalcemia?

Clinically evident hypocalcemia generally presents in milder forms and is usually the result of a chronic disease state. The patient may complain of muscle cramping, perioral or finger paresthesias, shortness of breath secondary to bronchospasm, and tetanic

contractions. Chronic hypocalcemia can present with cataracts, poor dentition, dry skin, coarse hair, and pruritus. Patients may have a Chvostek sign—facial or eye muscle twitching when the examiner taps the facial nerve. They may also have a Trousseau sign, manifested when the examiner inflates the blood pressure cuff to 20 mm Hg above the systolic blood pressure for 3 minutes and induces carpal spasms because of local ulnar and median nerve ischemia. The Trousseau sign is relatively specific for hypocalcemia, while the Chvostek test is less diagnostic.38 More severe signs and symptoms include hypotension, QT prolongation, angina, and congestive heart failure.

Most cases of hypocalcemia are discovered by clinical suspicion and appropriate laboratory testing on emergency department arrival. A serum calcium level less than 8.5 mg/dL or an ionized calcium level less than 2 mEq/L is considered abnormal. A whole blood ionized calcium should be performed rapidly to avoid changes due to chelation and pH.39 An ECG should be obtained, and cardiac monitoring should be initiated to rule out arrhythmias and a prolonged QT interval.

Most asymptomatic patients or those with mild symptoms may be treated with oral calcium supplementation such as calcium carbonate. Intravenous calcium can be administered either as calcium chloride or calcium gluconate. Most

Table 5.“Bones,moans,stones,andgroans”mnemonicforsymptomsthatshouldraisesuspicionofhypercalcemia

Bones Bone pain

Moans Psychic moans such as fatigue, lethargy, depression, confusion, nausea, lack of appetite

Stones Kidney stones

Groans Constipation

Table 6.Fivemostcommonsymptomaticcausesofhypocalcemiaseeninemergencydepartments

Alcohol abuse/chronic malnutrition

Toxins (hydrofluoric acid, ethylene glycol)

Pancreatitis (via saponification)

Tumor lysis syndrome

Massive blood transfusion (greater than 10 units)

Page 10: Volume 25 • Number 10 In This Issue · 2011-07-12 · Electrolyte abnormalities are common in emergency medicine, but the degree of severity can vary greatly. Asymptomatic electrolyte

June2011•Volume25•Number10

9

patients requiring intravenous calcium should be admitted to the hospital for monitoring and treatment of nausea, vomiting, hypertension, and bradycardia.

HypomagnesemiaThere are many causes of

hypomagnesemia including dietary, gastrointestinal, renal, endocrine/metabolic, and drug-induced causes. Emergency department patients at high risk for clinically significant hypomagnesemia include patients on diuretics, alcoholics and other chronically malnourished patients, patients with hypokalemia, patients with acute myocardial infarction, and patients with ventricular arrhythmias.40,41

Patients begin to manifest symptoms of hypomagnesemia at serum levels below 1.2 mEq/L, but symptoms do not always correlate with the total serum magnesium level. Most of the total body magnesium is intracellular, and therefore a single low serum magnesium level may not reflect the degree of total body hypomagnesemia.

A stable patient with hypomagnesemia may be treated with a loading dose of 1 to 2 grams of magnesium sulfate over 10 to 60 minutes, followed by a standard maintenance dose of ½ to 1 gram per hour until symptoms have resolved or a therapeutic effect is obtained.14 We routinely use 0.5 grams per hour in patients who are euvolemic and have normal renal function. It is important to remember that there are potential adverse effects to rapid, aggressive magnesium replacement at rates faster than 2 grams per hour, including decreased deep-tendon reflexes, respiratory depression, and heart block. Magnesium gluconate oral supplementation can be given if the patient is only mildly hypomagnesemic and asymptomatic.

HypermagnesemiaHypermagnesemia is a rare

electrolyte abnormality that is most often seen in patients with impaired

or no renal function and in patients receiving intravenous magnesium for medical treatment, especially when their hydration state and renal function are not taken into account.42 Magnesium is an ingredient in many over-the-counter laxatives and antacids as well.43 It can also be seen in eclamptic women being treated with high doses of magnesium. It is important to remember that magnesium is a central nervous system and neuromuscular depressant and can cause cardiac instability.

Most stable or asymptomatic patients may be treated with cessation of their magnesium therapy. In patients with higher concentrations of magnesium and renal failure, or more severe symptoms, renal consultation should be initiated immediately to arrange for dialysis. Intravenous calcium is an option for

reversal of magnesium toxicity, but this should be reserved for patients with life-threatening symptoms while dialysis is being arranged.14 Calcium is especially helpful in eclamptic patients who become acutely toxic when being treated with high-dose magnesium.

HypophosphatemiaHypophosphatemia, like

hypomagnesemia, often goes unrecognized. Although most patients remain asymptomatic, severe hypophosphatemia can result in potentially life-threatening complications. Patients typically begin to manifest symptoms of hypophosphatemia at serum levels below 1 mg/dL.44 The emergency department patients most likely to have hypophosphatemia are those who are malnourished with alcohol

Pearls• In patients with suspected hyperkalemia, calcium should be

administered without waiting for laboratory confirmation if the patient’s ECG shows widening QRS, because these patients can progress to life-threatening unstable rhythms within minutes.

• Low serum potassium levels reflect a much greater total potassium deficit; correction of large deficits can require several days.

• An ionized calcium level should be obtained for any patient with suspected hypocalcemia.

• Asymptomatic patients with hypocalcemia or those with mild symptoms may be treated with oral calcium supplementation such as calcium carbonate; however, in severe, symptomatic hypocalcemia, intravenous calcium may be administered, preferably as calcium gluconate or calcium chloride.

• Hypertonic saline should be reserved for hyponatremic patients with neurologic signs including coma, seizure, or focal deficit and with a serum sodium level below 120 mEq/L.

Pitfalls• Failing to administer at least 0.5 grams/hr of magnesium sulfate

along with potassium replacement in hypokalemic patients to ensure that the potassium moves into the cells.

• Rapidly correcting hyponatremia; serum sodium should be corrected at a maximal rate of 0.5 mEq/hr, taking care to raise the sodium level no more than 10 to 12 mEq/day to avoid central pontine myelinolysis.

• Rapidly correcting hypernatremia with hypotonic fluids; this can cause cerebral edema resulting in worsening mental status and death.

Page 11: Volume 25 • Number 10 In This Issue · 2011-07-12 · Electrolyte abnormalities are common in emergency medicine, but the degree of severity can vary greatly. Asymptomatic electrolyte

CriticalDecisionsinEmergencyMedicine

10

withdrawal and patients with diabetic ketoacidosis in whom re-introduction of insulin and glucose causes phosphate uptake into cells.

Severe hypophosphatemia can manifest as seizures, arrhythmias, cardiomyopathy, insulin resistance, rhabdomyolysis, and acute respiratory failure. The diagnosis of hypophosphatemia should always be considered in malnourished patients with impaired respiratory function with or without weakness. Therapy for hypophosphatemia is recommended as levels drop below 1.5 to 2 mg/dL and is mandatory as levels fall below 1 mg/dL.45 Since hypophosphatemia often presents with hypokalemia, phosphate repletion is often given with potassium repletion. Phosphorous can be orally repleted for stable or asymptomatic patients. Intravenous preparations should be slowly administered; we recommend 0.5 mL/hr of potassium phosphate (K2PO4) for all but the most severely symptomatic and hypophosphatemic patients. Infusion rates should never exceed 1 mL of potassium phosphate per hour.

HyperphosphatemiaHyperphosphatemia most often

occurs chronically in patients with renal failure, whereas acute phosphate overload is usually due to tissue breakdown or iatrogenic overdose. Hypoparathyroidism, tumor calcinosis, and bisphosphonate treatment can cause increased tubular reabsorption of phosphate by the kidneys.46

Because most patients presenting with severe hyperphosphatemia also have hypocalcemia, treatment should be focused on the correction of both electrolytes. Hyperphosphatemia causes hypocalcemia by precipitating calcium out of the blood and decreasing vitamin D production. It is this secondary hypocalcemia that can cause muscle cramping, tetany, and seizures. It is important to work to resolve the underlying disease state responsible for the imbalance.

Phosphate excretion can be increased by saline infusion, and acetazolamide should be considered. Hyperphosphatemia usually resolves in 6 to 12 hours in patients with normal renal function47; however, patients with hyperphosphatemia and hypocalcemia along with renal failure may require hemodialysis.

Case Resolutions

n Case OneOn recognizing life-threatening

hyperkalemia in this dialysis-dependent woman, the emergency physician ensured that intravenous access was immediately obtained. Based on the patient’s hyperkalemia with a wide QRS, severe acidemia, and renal failure, she was given one 10-mL ampule of 10% calcium chloride, 10 units of insulin (no glucose was immediately administered because the patient was already hyperglycemic), and one 50-mL ampule of sodium bicarbonate. Nephrology was consulted for emergent hemodialysis.

n Case TwoThe college student in this

case was suffering from severe hyponatremia likely secondary to use of the recreational drug N-methyl-3,4-methylenedioxyamphetamine (MDMA or ecstasy).48 On arrival, the patient was having seizure activity that was unresponsive to benzodiazepines. MDMA-induced hyponatremia occurs via multiple mechanisms; these include the induction of syndrome of inappropriate antidiuretic hormone (SIADH), the tendency among those who are intoxicated to be involved in vigorous activity, and the encouragement to drink large amounts of water to prevent dehydration and rhabdomyolysis.

The patient’s airway was established, breath sounds were confirmed, and large-bore intravenous access was obtained. The physician administered 100 mL of 3% hypertonic saline over 10 minutes followed by a second bolus of 100 mL of the 3% solution en route to

the ICU. The patient’s sodium was corrected over a time course of 48 to 72 hours.

SummaryElectrolyte abnormalities are very

common in emergency medicine practice and rarely occur in isolation. Emergency physicians must be able to identify common disease states that place patients at risk for serious electrolyte disturbances and understand the emergency treatments for each electrolyte abnormality. It is important to be able to identify the classic signs and symptoms of common electrolyte disturbances quickly. Although presentations that are asymptomatic may be slowly corrected, those that indicate profound mental status changes or life-threatening arrhythmias may require immediate correction to avoid cardiac arrest.

Most asymptomatic patients can be discharged home safely with good outpatient followup. However, any patient who presents with altered mental status, concerning ECG changes, significant electrolyte abnormalities, or serious underlying disease processes requires admission for further treatment and evaluation.

References1. AlfonzoAV,IslesC,GeddesC,DeighanC.Potassium

disorders—clinicalspectrumandemergencymanagement.Resuscitation.2006;70:10-25.

2. SlovisC,JenkinsR.ABCofclinicalelectrocardiography:conditionsnotprimarilyaffectingtheheart.BMJ.2002;324(7349):1320-1323.

3. AllonM,ShanklinN.Effectofbicarbonateadministrationonplasmapotassiumindialysispatients:interactionswithinsulinandalbuterol.Am J Kidney Dis.1996;28:508-514.

4. AckerCG,JohnsonJP,PalevskyPM,GreenbergA.Hyperkalemiainhospitalizedpatients:causes,adequacyoftreatment,andresultsofanattempttoimprovephysiciancompliancewithpublishedtherapyguidelines.Arch Intern Med.1998;158(8):917-924.

5. ParhamWA,MehdiradAA,BiermannKM,FredmanCS.Hyperkalemiarevisited.Tex Heart Inst J. 2006;33(1):40-47.

6. AhmedJ,WeisbergLS.Hyperkalemiaindialysispatients.Semin Dial.2001;14:348-356.

7. BlumbergA,WeidmannP,ShawS,GnädingerM.Effectofvarioustherapeuticapproachesonplasmapotassiumandmajorregulatingfactorsinterminalrenalfailure.Am J Med.1988;85(4):507-512.

8. CarvalhanaV,BurryL,LapinskySE.Managementofseverehyperkalemiawithouthemodialysis:casereportandliteraturereview.J Crit Care.2006;21(4):316-321.

9. AllonM,DunlayR,CopkneyC.Nebulizedalbuterolforacutehyperkalemiainpatientsonhemodialysis.Ann Intern Med.1989;110(6):426-429.

Page 12: Volume 25 • Number 10 In This Issue · 2011-07-12 · Electrolyte abnormalities are common in emergency medicine, but the degree of severity can vary greatly. Asymptomatic electrolyte

June2011•Volume25•Number10

11

10. MahoneyBA,SmithWA,LoDS,etal.Emergencyinterventionsforhyperkalaemia.Cochrane Database Syst Rev. 2005;(2):CD003235.

11. AllonM,CopkneyC.Albuterolandinsulinfortreatmentofhyperkalemiainhemodialysispatients.Kidney Int.1990;38:869-872.

12. Gruy-KapralC,EmmettM,SantaAnaCA,etal.Effectofsingledoseresin-cathartictherapyonserumpotassiumconcentrationinpatientswithend-stagerenaldisease.J Am Soc Nephrol.1998;9(10):1924–1930.

13. LinJL,LimPS,LeuML,HuangCC.Outcomesofseverehyperkalemiaincardiopulmonaryresuscitationwithconcomitanthemodialysis.IntensiveCare Med. 1994;20(4):287-290.

14. ECCCommittee,SubcommitteesandTaskForcesoftheAmericanHeartAssociation.2005AmericanHeartAssociationGuidelinesforCardiopulmonaryResuscitationandEmergencyCardiovascularCare.Circulation.2005;112:IV-121–IV-125.

15. WhangR,OeiTO,AikawaJK,etal.Predictorsofclinicalhypomagnesemia.Hypokalemia,hypophosphatemia,hyponatremia,andhypocalcemia.Arch Intern Med.1984;144:1794–1796.

16. CocaSG,PerazellaMA,BullerGK.Thecardiovascularimplicationsofhypokalemia.Am Journal Kidney Dis. 2005;45(2):233-247.

17. KrahnLE,LeeJ,RichardsonJW,etal.Hypokalemialeadingtotorsadesdepointes.Munchausen’sdisorderorbulimianervosa?Gen Hosp Psychiatry.1997;19:370-377.

18. GennariFJ.Hypokalemia.N Engl J Med.1998;339(7):451-458.

19. GrahamDY.Effectivenessandtoleranceof“solid”vs.“liquid”potassiumreplacementtherapy.In:CameronJS,GlussockRJ,WheltonA,eds.Kidney Disease.NewYork,NY:MarcelDekkerInc;1986.

20. MelikianAP,ChengLK,WrightGJ,etal.Bioavailabilityofpotassiumfromthreedosageforms:suspension,capsule,andsolution.J Clin Pharmacol.1988;28:1046-1050.

21. HamillRJ,RobinsonLM,WexlerHR,MooteC.Efficacyandsafetyofpotassiuminfusiontherapyinhypokalemiccriticallyillpatients.Crit Care Med.1991;613-617.

22. MorrillGB,KatzMD.Theuseoflidocainetoreducethepaininducedbypotassiumchlorideinfusion.J Intraven Nurs.1988;11(2):105-108.

23. WhangR,WhangDD,RyanMP.Refractorypotassiumrepletion.Aconsequenceofmagnesiumdeficiency.Arch Intern Med.1992;152:40-45.

24. UpadhyayA,JaberBL,MadiasNE.Incidenceandprevalenceofhyponatremia.Am J Med.2006;119(7Suppl1):S30-S35.

25. KumarS,BerlT.Sodium.Lancet.1998;352:220-228.

26. LaurenoR,KarpBI.Myelinolysisaftercorrectionofhyponatremia.Ann Intern Med.1997;126:57-62.

27. PirzadaNA,AliII.Centralpontinemyelinolysis.Mayo Clin Proc.2001;76:559-562.

28. HillierTA,AbbottRD,BarrettEJ.Hyponatremia:evaluatingthecorrectionfactorforhyperglycemia.Am J Med.1999;106(4):399-403.

29. OrenRM.Hyponatremiaincongestiveheartfailure.Am J Cardiol.2005;95(9A):2B-7B.

30. HewTD,ChorleyJN,CiancaJC,DivineJG.Theincidence,riskfactors,andclinicalmanifestationsofhyponatremiainmarathonrunners.Clin J Sport Med.2003;13:41–47.

31. BeckLH.Changesinrenalfunctionwithaging.Clin Geriatr Med.1998;14:199-209.

32. BarriYM,KnochelJP.Hypercalcemiaandelectrolytedisturbancesinmalignancy.Hematol Oncol Clin North Am.1996;10:775–790.

33. AriyanC,SosaJA.Assessmentandmanagementofpatientswithabnormalcalcium.Crit Care Med. 2004;32:S146-S154.

34. RuDuskyBM.ECGabnormalitiesassociatedwithhypocalcemia.Chest.2001;119:668-669.

35. LeGrandSB,LeskuskiD,ZamaI.Narrativereview:furosemideforhypercalcemia:anunprovenyetcommonpractice.Ann Intern Med.2008;149(4):259-263.

36. HurleyK,BaggsD.Hypocalcemiccardiacfailureintheemergencydepartment.J Emerg Med.2005;28(2):155-159.

37. ChavanCB,SharadaK,RaoHB,NarsimhanC.Hypocalcemiaasacauseofreversiblecardiomyopathywithventriculartachycardia.Ann Intern Med.2007;146(7):541-542.

38. UrbanoFL.Signsofhypocalcemia:Chvostek’sandTrousseau’s.Hosp Physician.2000;36:43-45.

39. DickersonRN,MorganLM,AlexanderKH,etal.Accuracyofmethodstoestimateionizedand“corrected”serumcalciumconcentrationsincriticallyillmultipletraumapatientsreceivingspecializednutritionsupport.JPEN.2005;28:133–141.

40. ElisafM,MerkouropoulosM,TsianosEV,SiamopoulosKC.Pathogenicmechanismsofhypomagnesemiainalcoholicpatients.J Trace Elem Med Biol.1995;9:210–214.

41. DaceyMJ.Hypomagnesemicdisorders.Crit Care Clin.2001;17:155–173.

42. MussoCG.Magnesiummetabolisminhealthanddisease.Int Urol Nephrol.2009;41(2):357-362.

43. QureshiT,MelonakosTK.Acutehypermagnesemiaafterlaxativeuse.Ann Emerg Med.1996;28(5):552-555.

44. JansonC,BirnbaumG,BakerFJ2nd.Hypophosphatemia.Ann Emerg Med.1983;12:107-116.

45. LentzRD,BrownDM,KjellstrandCM.Treatmentofseverehypophosphatemia.Ann Intern Med.1978;89:941-944.

46. WaltonRJ,RussellRG,SmithR.Changesintherenalandextrarenalhandlingofphosphateinducedbydisodiumetidronate(EHDP)inman.Clin Sci Mol Med.1975;49:45–56.

47. SlovisCM,MeehanPM.Electrolyteabnormalities.In:RoppoloL,DavisD,KellyS,RosenP,eds.Emergency Medicine Handbook: Critical Concepts for Clinical Practice.Philadelphia,PA:MosbyInc;2006:1166-1184.

48. HolmesSB,BanerjeeAK,AlexanderWD.Hyponatraemiaandseizuresafterecstasyuse.Postgrad Med J. 1999;75(879):32-33.

Page 13: Volume 25 • Number 10 In This Issue · 2011-07-12 · Electrolyte abnormalities are common in emergency medicine, but the degree of severity can vary greatly. Asymptomatic electrolyte

CriticalDecisionsinEmergencyMedicine

12

The LLSA Literature Review“The LLSA Literature Review” summarizes articles from ABEM’s “2012 Lifelong Learning and Self-Assessment Reading List.” These articles are available online in the ACEP LLSA Resource Center (www.acep.org/llsa) and on the ABEM Web site.

Article 2

Acute-Onset Floaters and Flashes: Is This Patient at Risk for Retinal Detachment?ReviewedbyJeffreyN.Siegelman,MD,andJ.StephenBohan,MD,MS,FACEP;HarvardAffiliatedEmergencyMedicineResidency;BrighamandWomen’sHospital

HollandsH,JohnsonD,BroxAC,etal.Acute-onsetfloatersandflashes:isthispatientatriskforretinaldetachment?JAMA.2009;302(20):2243-2249.

Emergency physicians treating patients presenting with acute-onset visual floaters or flashes must identify patients at increased risk of retinal detachment who require ophthalmo-logic evaluation most urgently.

Posterior vitreous detachment (PVD), the separation of the posterior vitreous from the retina, is the most common cause of visual floaters (gray or dark spots that move through the visual field) and flashes (unilateral fleeting bursts of light in the periphery). PVD is often associated with advanced age, trauma, myopia, or intraocular inflammation. The acute phase of PVD can also be associated with full-thickness reti-nal tears, leading to fluid in the subretinal space and retinal detachment.

Evaluation begins with a detailed history of the visual symptoms and a search for both ocular and nonocular causes. Visual aura associated with migraine can mimic PVD and is differentiated by a description of colored (as opposed to white) flashes, the association with a headache, and a normal ocular examination. More rare occipital disorders including ischemia, hemorrhage, neoplasm, and seizure should also be considered and ruled out with neurologic examination and testing as warranted.

Examination begins with testing visual acuity, corrected using the patient’s glasses or a pinhole, and continues with confrontation visual field testing to assess for an area of de-tached retina and assessment of pupil response to direct and consensual stimulation. Slit lamp examination may show vitreous pigment or hemorrhage, commonly referred to as “tobacco dust,” which represents melanin in the vitreous re-leased by a retinal tear. This is demonstrated by focusing the

slit lamp beam behind the lens into the anterior vitreous. The eyes are then dilated with a mydriatic agent (eg, one drop each of tropicamide 1% and phenylephrine 2.5%) and direct ophthalmoscopy is performed to detect an obvious detach-ment or hemorrhage. It is not necessary to measure intraocu-lar pressure in these patients.

Other ocular causes of floaters and flashes are retinal tear or detachment and posterior uveitis. If flashes predominate, consider oculodigital stimulation, rapid eye movements, or macular degeneration. If the patient primarily has floaters, vitreous hemorrhage from proliferative retinopathy is pos-sible. If an ocular cause is suspected, and without eye pain or photosensitivity (which suggests an inflammatory ocular condition), PVD is the presumed diagnosis.

If retinal detachment is suspected, emergency ophthalmo-logic consultation is warranted. One concerning symptom is a monocular visual field defect, sometimes described as a “curtain of darkness.”

A metaanalysis of patients with acute-onset floaters or flashes and diagnosed PVD demonstrated a prevalence of retinal tear of 14%. Rates of tears were similar in patients who had either floaters or flashes but not both. Subjective vision reduction had a five times greater association with retinal tear.

Patients with progressive unilateral visual field loss, sub-jective or objective visual reduction, or vitreous pigment or hemorrhage on examination require same-day ophthalmo-logic evaluation. Those with new floaters or flashes with-out high-risk features can be referred for specialist followup within 1 to 2 weeks and given anticipatory guidance. Patients with chronic PVD have a 3.4% incidence of tear and should be warned to seek care for a sudden increase in the number of floaters or a new decrease in vision.

Highlights• Theacuteonsetofvisualfloatersorflashescanindicate

aretinaltearordetachmentthatrequiresemergentophthalmologicconsultation.

• Monocularvisualfielddefect,subjectiveorobjectivevisualacuitydecrease,andabnormalslitlampexaminationresultsareallhigh-riskfindingsforpossibleretinaltear.

• Patientswithchronicfloatersorflashesneedtobealertforasuddenincreaseintheirsymptomsorchangeintheirvisualacuity.

Page 14: Volume 25 • Number 10 In This Issue · 2011-07-12 · Electrolyte abnormalities are common in emergency medicine, but the degree of severity can vary greatly. Asymptomatic electrolyte

June2011•Volume25•Number10

13

Sinus tachycardia, rate 123, low voltage, electrical alternans, findings diagnostic of large pericardial effusion. The ECG demonstrates findings that are diagnostic of large pericardial effusion: tachycardia, low voltage, and electrical alternans. Low voltage is associated with large pericardial effusions, myxedema, large pleural effusions, end-stage cardiomyopathy, severe chronic obstructive pulmonary disease, severe obesity, infiltrative myocardial diseases, constrictive pericarditis, and prior massive MI. New low voltage, especially in the presence of tachycardia, should strongly suggest the presence of a large pericardial effusion. Electrical alternans, variations in the amplitudes of the QRS complexes, is not specific for pericardial effusion. However, its presence in the setting of low voltage is highly specific for large pericardial effusions. Electrical alternans is presumed to be caused by the pendular motion of the heart within the fluid-filled pericardial sac. This patient had developed a large pericardial effusion with pericardial tamponade.

Feature Editor: Amal Mattu, MD, FACEP

From: Mattu A, Brady W. ECGs for the Emergency Physician. London: BMJ Publishing; 2003:112,145-146. Available at www.acep.org/bookstore. Reprinted with permission.

The Critical ECGA 54-year-old woman with metastatic breast cancer presents with shortness of breath; blood pressure is 85/45.

Page 15: Volume 25 • Number 10 In This Issue · 2011-07-12 · Electrolyte abnormalities are common in emergency medicine, but the degree of severity can vary greatly. Asymptomatic electrolyte

CriticalDecisionsinEmergencyMedicine

14

n ObjectivesOn completion of this lesson, you should be able to:

1. Outlineaplanfordealingwithindeterminateordiscordantcardiacbiomarkerresults.

2. Discussthekineticsofreleaseintotheserumofcommonlyusedcardiacbiomarkers.

3. Explainthesignificanceoftroponinelevationsinpatientswithrenalfailure.

4. Describethenon-coronaryarterydiseasecausesoftroponinelevations.

5. Describetheusefulnessofcardiacbiomarkersinnon-coronaryarterydiseaseprocesses.

6. Explaintheappropriateroleforcardiacbiomarkersinriskstratificationforacutecoronarysyndrome.

n From the EM Model1.0 Signs,Symptoms,andPresentations

1.3 ChestPain

AlexanderT.Limkakeng,Jr,MD,FACEP,andJesminEhlers,MD

Cardiac Troponin

Lesson 20

Cardiovascular disease remains a leading cause of death in the United States, and acute coronary syndrome (ACS), particularly, is concerning. Patients with ACS can present with a wide variety of symptoms, none of which is sufficient to rule in or rule out the disease.1 This makes it difficult to identify which patients need extensive evaluation. Physicians fear missing a myocardial infarction (MI) in the emergency department because it is a leading cause of medicolegal liability. On the other hand, vast resources are spent on ruling out ACS in patients who present with concerning symptoms.

Previous studies have demonstrated that the patient history, physical examination, and ECG are insufficient to identify a patient population that can be safely discharged from the emergency department.2,3 This challenge has led to the development of new technologies such as 15-lead ECGs, neural networks, emergency department stress testing, and computed tomography coronary angiography to improve identification of ACS in the emergency department.

Currently, however, serial ECG and serum cardiac biomarkers are the cornerstone of ACS evaluation. This is with good reason. Cardiac troponin elevations have been shown to predict which patients with ACS symptoms are at risk for major cardiac adverse events and therefore will benefit from aggressive treatment strategies.4 Cardiac biomarkers are relatively inexpensive, noninvasive,

reproducible, fast, and objective. They do not require specialty experts such as radiologists, cardiologists, or nuclear cardiologists to interpret.

As such, one would think that there are few issues surrounding the use of cardiac troponins. In reality, a number of clinical scenarios arise in which the correct interpretation of cardiac troponins becomes difficult. The emergency physician must be familiar with how troponins work and, just as with any other test, know how to interpret the results of these tests. As the number of options for ACS evaluation increases, correct interpretation of cardiac troponins becomes more important.

Case Presentations

n Case OneA 45-year-old man presents with

exertional substernal diffuse chest pressure radiating to both shoulders; it has bothered him intermittently for the past 2 days. The most recent episode of pain is the most severe and started while the patient was carrying boxes up the stairs just prior to his presentation. Past medical history is significant for hypertension and hypercholesterolemia.

The patient called for an ambulance and was given aspirin and three nitroglycerin tablets sublingually, which provided relief of his pain. His vital signs are blood pressure 165/87, pulse rate 85, respiratory rate 17, and temperature of 37.8°C (100°F). On physical examination, the patient is an obese, diaphoretic man who appears to be in

Page 16: Volume 25 • Number 10 In This Issue · 2011-07-12 · Electrolyte abnormalities are common in emergency medicine, but the degree of severity can vary greatly. Asymptomatic electrolyte

June2011•Volume25•Number10

15

mild discomfort. The examination is otherwise unremarkable.

His ECG shows borderline ST-segment depression in leads aVF, II, and III, with normal sinus rhythm. His myoglobin level is 214 mcg/L (reference range for normal is <110 mcg/L), creatine kinase-MB (CK-MB) is 14 mg/dL (reference range for normal is <12 mg/dL), and troponin I is 0.9 ng/mL (reference range 0.02-1.2 ng/mL). His chest radiograph is normal.

A repeat ECG is unchanged. When a cardiologist is consulted regarding disposition of this patient, he recommends serial markers and ECGs in the observation unit with stress testing the following day.

n Case TwoA 67-year-old man with end-stage

renal disease and who is on dialysis presents with chest pain. The pain is diffuse across his chest and feels like a pressure. The pain started as a twinge last night, then worsened this morning. He feels like he cannot get a full breath in and is winded even with walking across the room. He has had some nonproductive coughing. He missed his dialysis session yesterday because he said he felt too ill to go. He notes that his legs are swollen but at their baseline levels. His past medical history is significant for hypertension, diabetes, and high cholesterol, and he has a 20 pack-year history of smoking.

Physical examination shows an obese man in moderate distress from pain. Vitals signs are blood pressure 187/101, pulse rate 101, respiratory rate 22, temperature 37.9°C (100.2°F), and pulse oximetry 95% on room air.

Further examination reveals normal S1 and S2, 2/6 early systolic murmur, slightly labored breathing with slight bibasilar rales, nontender abdomen, 2+ bilateral lower extremity edema with good pulses, and mild jugular venous distention.

The ECG shows a rate of 101, sinus rhythm, and left ventricular hypertrophy with concave ST-segment elevations in leads V4-V6. The chest radiograph shows mild cardiomegaly and mild pulmonary edema, bilaterally. His electrolytes are notable for marked elevation of creatinine (7.9 mg/dL) and BUN (61 mg/dL), as well as potassium of 5.6 mg/dL. His first set of cardiac markers shows a creatine kinase level of 561 units/L (reference range normal <210 units/L), CK-MB of 12 mg/dL (reference range for normal is <12 mg/dL), and troponin T level of 1.2 ng/mL (reference range 0.01-0.1 ng/mL).

A cardiologist is consulted and recommends sending the patient to his nephrologist for dialysis and discharge, stating that renal failure patients always have false-positive troponin elevations.

n Case ThreeA 39-year-old woman presents

because she has had constant chest pain for the past 3 days and then a syncopal episode. The chest pain developed suddenly following a car drive across country. It is diffuse across her chest and worse with deep breaths or coughing. Her cough is nonproductive. She does not have fever, nausea, or vomiting. She takes oral contraceptives and smokes.

The physical examination is remarkable for blood pressure 100/60,

pulse rate 111, respiratory rate 26, temperature of 36.7°C (98.1°F), and pulse oximetry 92% on room air. The patient appears mildly uncomfortable and is slightly diaphoretic. The rest of the examination, including examination of the heart and lungs, is unremarkable.

The ECG shows sinus tachycardia, without ST-segment elevations. There are T-wave inversions in leads V1-V3. The chest radiograph is normal. Creatine kinase level is 124 U/L, CK-MB is 7 mg/dL, and troponin I level is 2.4 mg/mL (0.04 - 2 mg/mL). A computed tomography scan of the chest reveals a large pulmonary embolism just distal to the bifurcation of the pulmonary artery. The patient is initiated on heparin, and the hospitalist accepts the patient for admission to a general telemetry bed.

Cardiac MarkersCreatine kinase is an enzyme

found in many parts of the body and can be fractionated into three isoenzymes: MM, MB, and BB. CK-MB is present in high concentrations in the myocardium but is also present in the lungs, small intestine, uterus, prostate, and healthy skeletal muscle. In response to cardiac injury, levels of CK-MB start to increase from baseline levels in 4 to 8 hours, peak in 12 to 24 hours, and usually return to normal within 3 days.5

Troponin is a structural protein found in striated muscle that is responsible for calcium processing. There are unique structural features of the cardiac muscle version of troponin that distinguish it from skeletal muscle versions of troponin and allow for detection in the serum. Cardiac

• WhatisthesignificanceofdiscordancebetweenCK-MBbandandtroponinvalues?

• Whatisthebestapproachtothepatientwith“indeterminate”troponinelevations?

• Whatisthebestapproachtopatientswithrenalfailurewithincreasedtroponinlevels?

• Whatarethenon-coronaryarterydiseasecausesofelevatedtroponin?

• Whatisthesignificanceofelevatedtroponininthesettingofnon-coronaryarterydiseasepresentations?

Critical Decisions

Page 17: Volume 25 • Number 10 In This Issue · 2011-07-12 · Electrolyte abnormalities are common in emergency medicine, but the degree of severity can vary greatly. Asymptomatic electrolyte

CriticalDecisionsinEmergencyMedicine

16

troponin occurs in three forms: I, T, and C. Most of the troponin is complexed to the contractile apparatus. A small amount (3% to 6%) exists that is not structurally bound. This small amount has been termed the “cytosolic pool.” It is released almost immediately, is detectable within 6 hours of coronary occlusion, and remains in the bloodstream for up to 10 days.6

CRITICAL DECISIONWhat is the significance of discordance between CK-MB band and troponin values?

For many years, measurement of CK-MB was the gold standard for diagnosis of MI. Troponins have rightly supplanted CK-MB as the gold standard because of their higher specificity to cardiac tissue. However, one advantage that CK-MB and other biomarkers still have is that elevations are detectable in the blood 1 to 2 hours faster than troponin using traditional assays. However, due to recent changes in proposed cutoffs and national consensus guidelines and the emergence of highly sensitive troponin assays, some experts now argue that troponin elevations can be detected much earlier—within 2 to 3 hours of infarction. These experts argue that CK-MB should no longer have a role in ACS risk stratification and, indeed, some institutions are discontinuing the use of CK-MB.7 For physicians working at hospitals that still use CK-MB and who are not comfortable with the fact that troponin values at very low levels can be imprecise, CK-MB should be considered an early marker of infarction.

There is evidence to support this approach. Storrow et al evaluated the prognostic significance of discordant markers (when the results of two different markers conflict). Out of 8,769 eligible registry patients, CK-MB was positive but troponin negative in 4.9%. The odds ratio for ACS in these patients compared to patients for whom both markers were negative was 2.2 (95% confidence

interval 1.7-2.8).8 Thus, patients with isolated CK-MB elevations should be considered higher risk for ACS, particularly early in the patient presentation.

CRITICAL DECISIONWhat is the best approach to the patient with “indeterminate” troponin elevations?

The most recent Joint European Society of Cardiology/American College of Cardiology Committee defines myocardial infarction by elevation of troponins. They define such an elevation as a measurement “exceeding the 99th percentile of a reference control group.” This is a lower level than almost all of the manufacturer-reported cutoffs. Unfortunately, at such low troponin levels, the precision of measurements decreases to unacceptable levels for almost all currently used troponin assays. The Committee statement also mandates that the coefficient of variation (a measure of assay precision that varies with the troponin level) be less than 10% at the cutoff level9 (Table 1). The troponin level at which current assays achieve this mandated precision is higher than the aforementioned 99th percentile, creating an “indeterminate” zone between the Committee-recommended cutoff, the level at which the coefficient of variation is less than 10%, and the manufacturer-recommended cutoff. Needless to say, there is some uncertainty about what to do for patients with troponin levels in this range.

There have been several studies indicating that the risk associated with troponin elevation increases linearly above any detectable level. In one such study, the authors created a receiver operating curve for troponin using data from their own emergency department population and derived their own cutoff at which 99% of all ACS patients were identified. This derived cutoff level was lower than the manufacturer derived cutoff. They then determined the rates of cardiovascular events for patients with levels below normal (undetectable), below their cutoff, above their cutoff, and above the manufacturer cutoff. They found that the risk of adverse cardiovascular events essentially rose linearly in the “indeterminate” range between the manufacturer’s detectable and cutoff levels (Figure 1).12

Thus, cutoff values for “positive” and “negative” are necessarily artificial. Over the long term, it is necessary to ensure that there are no false-positives to prevent unnecessary interventions. However, for the emergency physician, it is probably more important to prevent any false-negatives—to avoid missing potential ACS. Keeping in mind that there are many reasons for chronic, low levels of troponin elevations, it is thus important to recognize the potential significance of troponin elevations that are below the manufacturer-recommended cutoffs. Particularly, early in the course of disease when levels might still be rising, patients with any detectable troponin should

Table 1.Comparisonofmanufacturer99thpercentileand10%coefficientofvariation(COV)cutoffs10,11

Manufacturer assay 99th percentile (ng/mL) 10% COV cutoff (ng/mL)

Abbott AxSYM 0.5 0.8

Beckman-Coulter Access 0.04 0.06

Biosite Triage 0.19 0.5

Dade Behring Stratus CS 0.07 0.06

i-STAT 0.08 0.1

Boche Elecsys 0.01 0.035

Page 18: Volume 25 • Number 10 In This Issue · 2011-07-12 · Electrolyte abnormalities are common in emergency medicine, but the degree of severity can vary greatly. Asymptomatic electrolyte

June2011•Volume25•Number10

17

be considered higher risk unless there is another explanation for their levels.

Recently, high-sensitivity troponin assays have been introduced that can detect troponin at levels that are orders of magnitude lower than those of current-generation assays. The role for these new assays in clinical care has not yet been definitively established.

Renal Failure and Cardiac Biomarkers

The management of troponin elevations in patients with renal failure has been a source of controversy. In general, patients with chronic kidney disease are at almost six times higher risk for early death than those without renal disease, and many of these deaths are from cardiac causes.13 However, patients with renal failure often demonstrate a chronic troponin elevation even in the absence of any identifiable coronary disease.14 Since there is risk associated with aggressive intervention in these patients, false-positive troponin tests are potentially dangerous.

There is a common misperception that all troponin elevations in renal failure patients are false positives. Troponin is not renally cleared and therefore not affected by dialysis. Rather it is hypothesized that chronic troponin elevations in renal failure

are due to ventricular hypertrophy, chronic fluid overload, or endothelial dysfunction.6 In fact, even in asymptomatic outpatients with renal failure, the identification of chronic troponin elevation is associated with poorer long-term outcome.15 In one metaanalysis of such studies, troponin T but not troponin I elevation was associated with increased mortality in 12 to 24 months.16

Unfortunately, there is a relative dearth of high-quality clinical studies informing clinicians on how to rapidly differentiate a baseline chronic troponin elevation from an acute cardiac event. One sub-analysis of the large multicenter GUSTO IV trial revealed that patients with elevated troponin T levels were at a higher risk for adverse cardiac events and death in 30 days regardless of their renal function.17 Kontos and colleagues18 likewise found that troponin elevations were associated with increased mortality regardless of kidney function in 3,774 consecutive patients being admitted to the hospital for potential cardiac disease. On the other hand, Van Lente et al19 found peak troponin T levels to have sensitivity and specificity of 45% and 72%, respectively in the renal disease group. Peak troponin I levels had a sensitivity and specificity of 21% and 89%, respectively. These values

are markedly lower than those for patients without renal disease.

CRITICAL DECISIONWhat is the best approach to patients with renal failure with increased troponin levels?

The observation that elevated troponin and underlying renal disease are both independently associated with worse cardiovascular outcome needs to be balanced with an acknowledgment of the phenomena of chronic elevations and the risk of intervention. It is important to recall that the troponin test is one of several modalities of risk stratification. Therefore, troponin levels need to be interpreted in the context of the patient’s other clinical risk indicators. Given renal disease patients’ underlying increased risk, it is best to take a more conservative approach with regard to admission for serial markers and ECGs. Knowing the trend of troponin elevation can be useful. One available strategy is simply to repeat the test after several hours. The sensitivity and specificity of troponin tests increase with serial testing.10 It is also recommended, whenever possible, to determine whether the patient’s current level is an increase over a chronic baseline elevation.

CRITICAL DECISIONWhat are the non-coronary artery disease causes of elevated troponin?

Since the introduction of troponin assays, it has been observed that non-coronary artery diseases are associated with troponin elevations. Initially, there was some confusion as to whether this represented secondary ischemia from the primary disease process or an effect of the primary disease itself. It now is clear that troponin levels reflect the damage caused by disease processes other than acute cardiac ischemia.20 Table 2 shows a partial list of the many conditions associated with serum troponin elevations. In the next section, we will discuss those causes that are commonly seen in the emergency department.

Figure 1.Cardiovasculareventratesinpatientswithtroponinlevelsinthe“indeterminate”zone12

“Normal” <99th < Cutoff Standard troponin percentile 0.03- 0.067- cutoff <0.03 mcg/L 0.066 mcg/L 0.099 mcg/L >0.1 mcg/L

Page 19: Volume 25 • Number 10 In This Issue · 2011-07-12 · Electrolyte abnormalities are common in emergency medicine, but the degree of severity can vary greatly. Asymptomatic electrolyte

CriticalDecisionsinEmergencyMedicine

18

CRITICAL DECISIONWhat is the significance of elevated troponin in the setting of non-coronary artery disease presentations?

In pulmonary embolism (PE), the sudden increase in pulmonary artery pressure is hypothesized to lead to right ventricular overload and cardiac cell injury. It has been shown that troponin elevations correlate with the size of the PE as well as with clinical indicators such as severe hypoxemia, prolonged hypotension, cardiogenic shock, and the need for inotropic therapy or mechanical ventilation.21 A metaanalysis demonstrated that patients with confirmed PE and troponin elevations had a 30-day mortality rate of 19.7% compared to a rate of 3.7% for those with confirmed PE and normal troponin levels. This was true even for those with a normal blood pressure on admission.22

Troponin elevations have also been observed in patients with congestive heart failure (CHF), both during acute exacerbations and at baseline. As is the case for patients with renal failure, it is thought that troponin elevations in CHF are due to chronic fluid overload and that they identify patients with a worse long-term prognosis. In particular, they identify those with acute CHF exacerbations with worse prognosis.23

Patients with sepsis and a wide variety of other critical illnesses demonstrate troponin elevations. There is debate over which of the many proposed mechanisms is responsible, including regional wall motion abnormalities, microvascular thrombi, and direct toxic effect of endotoxins or inflammatory reactants.24 It has also been shown that these elevations correlate with higher mortality rates, even when controlling for preexisting cardiac disease.25

It can be seen, then, that troponin elevations are present in a wide spectrum of non-coronary artery disease processes. The key is recognizing that such elevations do not always reflect cardiac ischemia,

but rather identify patients who may need more aggressive intervention and monitoring for their primary disease process. Once again, other tools for determining risk for ACS should be used in conjunction with markers to determine if the patient is suffering from both ACS and another disease process.

Case Resolutions

n Case OneThe emergency physician

attending the 45-year-old man with exertional chest pain disagreed with the recommendation from the cardiology consultant. The emergency physician insisted that the cardiologist examine the patient in the emergency department before the patient was sent to the observation unit. After seeing the patient in the emergency department, the cardiologist admitted the patient to the cardiac ICU instead, where the patient’s second set of cardiac markers showed markedly elevated levels. The patient was started on antithrombotic medications in preparation for cardiac catheterization the next day. The angiogram revealed diffuse three-vessel disease with more than 70%

stenosis. The patient underwent successful coronary artery bypass graft surgery and had an uneventful hospital course.

n Case TwoThe 67-year-old man with renal

disease was sent to a dialysis center for treatment. He continued to feel worse, and 3 hours later he became hypotensive and increasingly dyspneic. An ambulance was called, and the patient was noted to decompensate, with alteration of mental status and evolution of sinus bradycardia into ventricular fibrillation. The paramedics started cardiopulmonary resuscitation and had given a first shock as they arrived in the emergency department. There, advanced cardiac life support protocols continued; the patient was intubated and was shocked a second time. The patient went into pulseless electrical activity arrest but responded to epinephrine, atropine, and bicarbonate, returning to sinus bradycardia. A repeat troponin level was 5.6 mg/mL. The patient was admitted to the ICU in unstable condition on vasopressor drugs, and hypothermia was initiated. He was

Table 2.Non-coronaryarterydiseasecausesoftroponinelevations6

Acute neurologic disease (including cerebrovascular accident, subarachnoid hemorrhage)

Aortic valve disease

Burns, especially if more than 30% total body surface area is involved

Cardiomyopathy (including hypertrophic obstructive cardiomyopathy, left ventricular hypertrophy)

Congestive heart failure (acute and chronic)

Drug toxicity (doxorubicin, fluorouracil, trastuzumab, snake venoms)

Hypertension

Hypotension

Hypothyroidism

Infiltrative diseases (amyloidosis, hemochromatosis, sarcoidosis, and scleroderma)

Inflammatory diseases (myocarditis, pericarditis, Kawasaki disease)

Pulmonary embolism, severe pulmonary hypertension

Renal failure

Sepsis

Trauma (including blunt contusion, ablation, pacing, internal cardioverter-defibrillator firings, endomyocardial biopsy, cardiac surgery)

Page 20: Volume 25 • Number 10 In This Issue · 2011-07-12 · Electrolyte abnormalities are common in emergency medicine, but the degree of severity can vary greatly. Asymptomatic electrolyte

June2011•Volume25•Number10

19

Pearls • Use serial testing of cardiac

markers whenever possible.

• Compare current biomarker levels with previous values, if they are available, to identify patients with chronic baseline elevations.

• CK-MB and myoglobin may still have a role in identifying early ACS.

• The precision of troponin assay results is reduced at very low levels of troponin; however, even minor elevations are associated with a higher rate of cardiac events and death.

• Knowledge of test characteristics is sometimes required to risk stratify patients.

Pitfalls• Relying on cardiac biomarkers

and failing to consider the clinical context.

• Assuming that troponin elevation is always ACS.

• Ascribing an elevated troponin result in patients with chronic renal insufficiency to their renal disease without consideration of other possibilities.

believed to be too unstable for cardiac catheterization, and medical therapy was pursued. Two days later, in light of the patient’s continued decline, the patient’s family elected to withdraw aggressive measures, and the patient died.

n Case ThreeThe 39-year-old woman with

the large pulmonary embolism was admitted to the floor still tachycardic. Over the next few hours she became increasingly tachypneic and her blood pressure dropped. She was

transferred rapidly to the ICU where an echocardiogram revealed right ventricular dilation and hypokinesis. The patient was given thrombolytic therapy, and her condition improved over the next several days. She was transferred to the floor, and discharged 7 days later. When the case was raised in quality assurance committee, the question arose as to whether there were any clinical indicators of potential deterioration.

Summary Cardiac biomarkers are an integral

part of the risk stratification of ACS. Given this role, it is important for emergency physicians to be very familiar with their use and limitations. In particular, emergency physicians must be wary of patients with discordant or “indeterminate” markers. They also must use other factors such as the history and physical examination when assessing patients with potentially confounding conditions such as a history of renal failure or congestive heart failure. It is important not only to interpret marker levels in clinical context, but also to use past levels and serial markers to help make decisions in difficult cases.

References 1. GoodacreS,LockerT,MorrisF,CampbellS.How

usefulareclinicalfeaturesinthediagnosisofacute,undifferentiatedchestpain?Acad Emerg Med.2002;9(3):203-208.

2. GoldmanL,CookEF,BrandDA,etal.Acomputerprotocoltopredictmyocardialinfarctioninemergencydepartmentpatientswithchestpain.N Engl J Med.1988;318(13):797-803.

3. LimkakengAJr,GiblerWB,PollackC,etal.CombinationofGoldmanriskandinitialcardiactroponinIforemergencydepartmentchestpainpatientriskstratification.Acad Emerg Med. 2001;8(7):696-702.

4. HeeschenC,GoldmannBU,TerresW,HammCW.CardiovascularriskandtherapeuticbenefitofcoronaryinterventionsforpatientswithunstableanginaaccordingtothetroponinTstatus.Eur Heart J. 2000;21(14):1159-1166.

5. AdamsJE3rd,MiracleVA.Cardiacbiomarkers:past,present,andfuture.Am J Crit Care.1998;7(6):418-423.

6. JaffeAS.Useofbiomarkersintheemergencydepartmentandchestpainunit.Cardiol Clin.2005;23(4):453-465.

7. SaengerAK,JaffeAS.Requiemforaheavyweight:thedemiseofcreatinekinase-MB.Circulation.2008;118(21):2200-2206.

8. StorrowAB,LindsellCJ,HanJH,etal.Discordantcardiacbiomarkers:frequencyandoutcomesinemergencydepartmentpatientswithchestpain.Ann Emerg Med.2006;48(6):660-665.

9. AlpertJS,ThygesenK,AntmanE,BassandJP.Myocardialinfarctionredefined—aconsensusdocumentofTheJointEuropeanSocietyofCardiology/AmericanCollegeofCardiologyCommitteefortheredefinitionofmyocardialinfarction.J Am Coll Cardiol.2000;36(3):959-969.

10. FesmireFM,DeckerWW,DiercksDB,etal.Clinicalpolicy:criticalissuesintheevaluationandmanagementofadultpatientswithnon-ST-segmentelevationacutecoronarysyndromes.Ann Emerg Med.2006;48(3):270-301.

11. AppleFS,ParvinCA,BuechlerKF,etal.Validationofthe99thpercentilecutoffindependentofassayimprecision(CV)forcardiactroponinmonitoringforrulingoutmyocardialinfarction.Clin Chem.2005;51(11):2198-2200.

12. ZarichSW,BradleyK,MayallID,BernsteinLH.MinorelevationsintroponinTvaluesenhanceriskassessmentinemergencydepartmentpatientswithsuspectedmyocardialischemia:analysisofnoveltroponinTcut-offvalues.Clin Chim Acta.2004;343(1-2):223-229.

13. GoAS,ChertowGM,FanD,etal.Chronickidneydiseaseandtherisksofdeath,cardiovascularevents,andhospitalization.N Engl J Med.2004;351(13):1296-1305.

14. LambEJ,WebbMC,AbbasNA.ThesignificanceofserumtroponinTinpatientswithkidneydisease:areviewoftheliterature.Ann Clin Biochem.2004;41(1):1-9.

15. AppleFS,MurakamiMM,PearceLA,HerzogCA.PredictivevalueofcardiactroponinIandTforsubsequentdeathinend-stagerenaldisease.Circulation.2002;106(23):2941-2945.

16. KhanNA,HemmelgarnBR,TonelliM,etal.PrognosticvalueoftroponinTandIamongasymptomaticpatientswithend-stagerenaldisease:ameta-analysis.Circulation.2005;112(20):3088-3096.

17. AvilesRJ,AskariAT,LindahlB,etal.TroponinTlevelsinpatientswithacutecoronarysyndromes,withorwithoutrenaldysfunction.N Engl J Med.2002;346(26):2047-2052.

18. KontosMC,GargR,AndersonFP,etal.OutcomesinpatientsadmittedforchestpainwithrenalfailureandtroponinIelevations.Am Heart J.2005;150(4):674-680.

19. VanLenteF,McErleanES,DeLucaSA,etal.Abilityoftroponinstopredictadverseoutcomesinpatientswithrenalinsufficiencyandsuspectedacutecoronarysyndromes:acase-matchedstudy.J Am Coll Cardiol.1999;33(2):471-478.

20. RoongsritongC,WarraichI,BradleyC.Commoncausesoftroponinelevationsintheabsenceofacutemyocardialinfarction:incidenceandclinicalsignificance. Chest.2004;125(5):1877-1884.

21. GiannitsisE,Müller-BardorffM,KurowskiV,etal.IndependentprognosticvalueofcardiactroponinTinpatientswithconfirmedpulmonaryembolism.Circulation.2000;102(2):211-217.

22. BecattiniC,VedovatiMC,AgnelliG.Prognosticvalueoftroponinsinacutepulmonaryembolism:ameta-analysis.Circulation.2007;116(4):427-433.

23. PernaER,MacinSM,ParrasJI,etal.CardiactroponinTlevelsareassociatedwithpoorshort-andlong-termprognosisinpatientswithacutecardiogenicpulmonaryedema. Am Heart J.2002;143(5):814-820.

24. MaederM,FehrT,RickliH,AmmannP.Sepsis-associatedmyocardialdysfunction:diagnosticandprognosticimpactofcardiactroponinsandnatriureticpeptides.Chest.2006;129(5):1349-1366.

25. ScottEC,HoHC,YuM,etal.Pre-existingcardiacdisease,troponinIelevationandmortalityinpatientswithseveresepsisandsepticshock.Anaesth Intensive Care.2008;36(1):51-59.

Page 21: Volume 25 • Number 10 In This Issue · 2011-07-12 · Electrolyte abnormalities are common in emergency medicine, but the degree of severity can vary greatly. Asymptomatic electrolyte

CriticalDecisionsinEmergencyMedicine

20

The Critical Image

A B

Retained gallstones within the biliary tree can cause biliary obstruction and pancreatitis, sometimes presenting in a delayed fashion after cholecystectomy. Ultrasonography may demonstrate ductal stones or dilated ducts resulting from obstruction, but a normal ultrasound does not exclude the diagnosis.

MRCP is the noninvasive test of choice for biliary ductal obstruction. On T2-weighted MR images, bile appears bright white allowing visualization of the intra- and extrahepatic bile ducts. Intraductal stones appear dark. MRCP has a reported sensitivity of 96% and specificity of 94% for biliary duct stones.1 A liver-specific gadolinium-based contrast agent such as gadoxetate disodium may also be given. This agent is taken up selectively by hepatocytes resulting in increased liver signal intensity. It is excreted in bile; biliary duct obstruction results in delayed excretion visible on delayed MR images.

CT may demonstrate dilated bile ducts and sometimes intraductal stones. Bile does not enhance with IV contrast and appears a dark color on CT soft tissue windows. CT protocols without any contrast and with special orally-ingested contrast agents excreted into bile have been tested. Sensitivity and specificity for these protocols are reported as 65%/84% and 92%/92%, respectively.1 CT with IV contrast is reported to have sensitivity and specificity for biliary stones near 85%—when radiologists are specifically cued to look for stones.2

An endoscopic retrograde cholangiopancreatography confirmed multiple biliary duct stones. These were removed, and sphincterotomy of the sphincter of Oddi was performed. She recovered uneventfully.1. SotoJA,AlvarezO,MuneraF,etal.Diagnosingbileductstones:comparisonofunenhancedhelicalCT,oralcontrast-enhancedCTcholangiography,andMRcholangiography. AJR Am J

Roentgenol. 2000;175(4):1127-1134.2. AndersonSW,LuceyBC,VargheseJC,etal.AccuracyofMDCTinthediagnosisofcholedocholithiasis.AJR Am J Roentgenol.2006;187(1):174-180.

Feature Editor: Joshua S. Broder, MD, FACEP

A 25-year-old woman presenting with epigastric abdominal and back pain and 3 days of vomiting. She was afebrile with normal vital signs. She had undergone laparoscopic cholecystectomy for symptomatic cholelithiasis 6 weeks earlier. Following her surgery, she had been asymptomatic for about 3 weeks before developing episodic abdominal pain. An outpatient right upper quadrant sonogram, performed the day before the current visit, was normal. At the current visit, the patient had an elevated lipase (719 U/L), a bilirubin of 4.6 mg/dL, and elevated alkaline phosphatase, ALT, and AST. She denied alcohol use. Her WBC count was normal. Magnetic resonance cholangiopancreatography (MRCP) was performed to evaluate for suspected biliary duct obstruction.

MorecaudadaxialsliceshowstheCBDenteringtheduodenum.AstoneobstructstheCBDatthislevelinthesphincterofOddi.

T2-weightedaxialMRimagethroughtheproximalcommonbileduct(CBD)showsadilatedCBD.

Duodenum Dilated common bile duct (8 mm) Stone lodged at sphincter of OddiDuodenum

Page 22: Volume 25 • Number 10 In This Issue · 2011-07-12 · Electrolyte abnormalities are common in emergency medicine, but the degree of severity can vary greatly. Asymptomatic electrolyte

June2011•Volume25•Number10

21

Qualified, paid subscribers to Critical Decisions in Emergency Medicine may receive CME certificates for up to 5 ACEP Category I credits, 5 AMA PRA Category I Credits™, and 5 AOA Category 2-B credits for answering the following questions. To receive your certificate, go to www.acep.org/criticaldecisionstesting and submit your answers online. You will immediately receive your score and printable CME certificate. You may submit the answers to these questions at any time within 3 years of the publication date. You will be given appropriate credit for all tests you complete and submit within this time. Answers to this month’s questions will be published in next month’s issue.

CME Questions

1. A 40-year-old man presents because for the past 3 days he has been weak and nauseated. On initial evaluation, he has stable vital signs but is found to have a serum potassium level of 6.8 mEq/L. Which of the following studies dictates the next step in management?A. 12-leadECGB. arterialbloodgasC. bloodglucoseD. chestradiographE. serumcreatinine

2. A young woman with glomerulonephritis and acute renal failure complains of weakness and palpitations. Her 12-lead ECG shows peaked T waves, absent P waves, and a QRS duration of 240 milliseconds. Which of the following is the next step in emergency department management?A. calciumgluconateB. furosemideC. hemodialysisD. insulinwithglucoseE. nebulizedalbuterol

3. A 40-year-old pregnant woman thought to have preeclampsia is transferred to the emergency department on a magnesium sulfate drip. She is noted to be lethargic, with respiratory depression. On examination, she has diffuse crackles in her lungs and absent reflexes. After providing oxygen and attaching a monitor, what the best treatment?A. activatedcharcoalB. calciumgluconateC. glucagonD. naloxoneE. sodiumbicarbonate

4. For which of the following patients would it be most appropriate to supplement potassium to keep serum levels above 4 mEq/L?A. a23-year-oldwomanwithpalpitationsandnopastmedical

historyB. a42-year-oldwomanwithpoorlycontrolleddiabetesmellitusC. a51-year-oldmanwithhypertension,recentlystartedon

lisinoprilD. a55-year-oldmanwithchestpainandprematureventricular

complexesE. a68-year-old-manwithchronicbronchitisandfever

5. Paramedics bring in a 42-year-old alcoholic man found wandering near a bus stop. He appears disheveled and malnourished. Approximately 4 hours after receiving intravenous glucose and thiamine, he complains of severe muscle aches and diffuse weakness. Supplementation with which of the following might have prevented his new symptoms?A. calciumgluconateB. diazepamC. normalsalineD. potassiumphosphateE. sodiumbicarbonate

6. A woman with diabetes is receiving normal saline and insulin for suspected diabetic ketoacidosis. Two hours later, she complains of severe, diffuse muscle weakness. Her cardiac monitor shows broad T waves, U waves, and frequent premature ventricular complexes. Supplementation with which of the following might have prevented this presentation?A. calciumgluconateB. dextroseC. potassiumchlorideD. sodiumbicarbonateE. sodiumphosphate

7. A 58-year-old man with history of small-cell lung cancer is brought in for weakness and pleuritic chest pain. He is alert, with normal vital signs, and has a nonfocal neurologic examination and a serum sodium of 129 mEq/L. What is the appropriate management of his hypernatremia?A. freewaterrestrictionB. hypertonicsalinebolusC. intravenousfurosemideD. normalsalinebolusE. sodiumchloridetablets

8. A 45-year-old woman presents with a 1-day history of cramping in the legs and forearms. She is one week postoperative from a thyroidectomy for follicular carcinoma. When her nurse inflates the blood pressure cuff, her ipsilateral hand contorts into a rigid fist. What is the treatment of choice?A. calciumgluconateB. diphenhydramineC. levothyroxineD. lorazepamE. magnesiumsulfate

Page 23: Volume 25 • Number 10 In This Issue · 2011-07-12 · Electrolyte abnormalities are common in emergency medicine, but the degree of severity can vary greatly. Asymptomatic electrolyte

CriticalDecisionsinEmergencyMedicine

22

9. A marathon runner is brought in after he collapsed at the 15th mile marker. He had been hydrating at water fountains placed every mile along the route. Suddenly, he begins to seize and does not stop despite 10 mg of diazepam. What is the most appropriate treatment?A. 5%dextrosesolutionB. fosphenytoinC. hypertonicsalineD. lorazepamE. pentobarbital

10. A 74-year-old woman with diabetes is brought from her nursing home for altered mental status. She is lethargic, with thready peripheral pulses, cool extremities, dry mucous membranes, and clear breath sounds. Her serum sodium level is 162 mEq/L with a blood glucose of 830 mg/dL. What is the best treatment?A. half-normalsaline,250mL/hrB. half-normalsalinebolus,1literC. intravenousinsulin,10unitsD. normalsaline,250mL/hrE. normalsalinebolus,1liter

11. Which of the following is the most accurate statement regarding cardiac markers and renal disease?A. alltroponinelevationsinrenalfailurepatientsarefalse

positivesB. anytroponinelevation,regardlessofrenalfunction,shouldbe

consideredsecondarytocoronaryarterydiseaseC. baselinetroponinlevelsandtrendsarehelpfulinthe

managementofpatientswithrenaldiseaseD. troponinelevationinrenalfailurepatientshasnoassociation

withmortalityE. troponinisrenallyclearedfromtheserum

12. Which of the following has been associated with a non-coronary artery disease cause of troponin elevations?A. abdominalaorticaneurysmB. hyperthyroidismC. hypothyroidismD. pneumoniaE. pneumothorax

13. Which of the following is true regarding the utility of cardiac biomarkers in non-coronary artery disease processes?A. CHFpatientswithchronictroponinelevationshaveaworse

long-termprognosisthandothosewithouttroponinelevationsB. inpatientswithpulmonaryemboli,troponinelevationshave

noassociationwithmortalityifthepatient’sbloodpressureisnormalatadmission

C. insepticpatients,ithasbeenclearlydemonstratedthattroponinelevationsareduetomicrovascularthrombi

D. thereisnocorrelationwithtroponinelevationsandthesizeofthepulmonaryemboli

E. troponinelevationsinthesettingofnon-coronaryarterydiseaseprocesseshavenoroleinclinicaldecisionmaking

14. Approximately how long do troponin levels remain elevated in the serum after cardiac injury?A. 18hoursB. 3daysC. 5daysD. 1weekE. 10days

15. Approximately how soon after cardiac injury is CK-MB usually detectable in the serum?A. within1hourB. 1-3hoursC. 2-4hoursD. 4-6hoursE. 6-8hours

16. How long do CK-MB levels remain elevated in the serum after cardiac injury?A. 18hoursB. 3daysC. 5daysD. 1weekE. 10days

17. Besides the myocardium, where else in the body is CK-MB found?A. boneB. brainC. hairD. skeletalmuscleE. skin

18. According to a large study by Storrow et al, which of the following is true regarding discordant markers?A. thebroadmajorityofpatientswithmyocardialinfarctionhave

discordantmarkersB. CK-MBisthecurrentgoldstandardfordeterminationofACSC. anelevatedCK-MBintheabsenceofelevatedtroponinis

associatedwithincreasedriskofadverseoutcomeD. renalfailureincreasesthelikelihoodofdiscordantmarkersE. troponinelevationsoccurmorerapidlyintheserumthan

CK-MBelevations

19. Which is true of indeterminate-level troponin elevations?A. theACC/ESCguidelinesrecommendusingmanufacturer-

recommendedcutoffsfordeterminationofMIB. theyareneverassociatedwithanyadverseoutcomesC. theyarethelevelsatwhichtroponinlevelsaremostpreciseD. theymayrepresentchronicbaselineelevationsE. theyshouldnotbeusedforroutinemanagementofthe

patientwithpotentialACS

20. Which of the following is the best response to the stable patient with potential ACS and indeterminate-level troponin elevations?A. administerthrombolyticsB. calculatethepatient’sestimatedglomerularfiltrationrateC. callcardiologyforastatcatheterizationD. orderastatechocardiogramE. trytodetermineifthepatienthaschronicbaselineelevations

Page 24: Volume 25 • Number 10 In This Issue · 2011-07-12 · Electrolyte abnormalities are common in emergency medicine, but the degree of severity can vary greatly. Asymptomatic electrolyte

June2011•Volume25•Number10

23

Answer key for May 2011, Volume 25, Number 9

1 2 3 4 5 6 7 8 9 10 11 12 13 14 15 16 17 18 19 20C C B A E B E A B E E E D C C C B D C E

The American College of Emergency Physicians (ACEP) makes every effort to ensure that its authors and editors are knowledgeable content experts. Readers are nevertheless advised that the statements and opinions expressed in this publication are provided as recommendations at the time of publication and should not be construed as official College policy. ACEP is not responsible for, and expressly disclaims all liability for, damages of any kind arising out of use, reference to, reliance on, or performance of such information. The materials contained herein are not intended to establish policy, procedure, or a standard of care. To contact ACEP, write to: PO Box 619911, Dallas, TX 75261-9911; call toll-free 800-798-1822, or 972-550-0911.

Change to CME ProcessesBeginning July 1, 2011, qualified subscribers who take the monthly posttest and submit their answers must make a score of 70% or better in order to receive a CME certificate.

Page 25: Volume 25 • Number 10 In This Issue · 2011-07-12 · Electrolyte abnormalities are common in emergency medicine, but the degree of severity can vary greatly. Asymptomatic electrolyte

NONPROFIT

U.S. POSTAGE

P A I D

DALLAS, TX

PERMIT NO. 1586

The Drug Box

June2011 • Volume25•Number10Critical Decisions in Emergency Medicine is the official CME publication of the American College of Emergency Physicians. Additional volumes are available to keep emergency medicine professionals up-to-date on relevant clinical issues.

Editor-in-ChiefLouis G. Graff IV, MD, FACEP

Professor of Traumatology and Emergency Medicine, Professor of Clinical Medicine, University of

Connecticut School of Medicine; Farmington, Connecticut

Section EditorJ. Stephen Bohan, MS, MD, FACEP

Executive Vice Chairman and Clinical Director, Department of Emergency Medicine, Brigham &

Women’s Hospital; Instructor, Harvard Medical School,Boston, Massachusetts

Feature EditorsMichael S. Beeson, MD, MBA, FACEP

Program Director, Department of Emergency Medi-cine, Akron General, Akron, Ohio; Professor, Clinical Emergency Medicine, Northeastern Ohio Universities

College of Medicine, Rootstown, Ohio

Joshua S. Broder, MD, FACEP Associate Clinical Professor of Surgery, Associate

Residency Program Director, Division of Emergency Medicine, Duke University Medical Center, Durham,

North Carolina

Amal Mattu, MD, FACEPProgram Director, Emergency Medicine Residency

Professor of Emergency MedicineUniversity of Maryland School of Medicine

Baltimore, Maryland

Associate EditorsDaniel A. Handel, MD, MPH, FACEP

Director of Clinical Operations, Department of Emergency Medicine, Oregon Health & Science University, Portland,

Oregon

Frank LoVecchio, DO, MPH, FACEPResearch Director, Maricopa Medical Center Emergency

Medicine Program; Medical Director, Banner Poison Control Center, Phoenix, Arizona; Professor, Midwestern

University/Arizona College of Osteopathic Medicine, Glendale, Arizona.

Sharon E. Mace, MD, FACEPAssociate Professor, Department of Emergency

Medicine, Ohio State University School of Medicine; Faculty, MetroHealth Medical Center/Cleveland Clinic Foundation Emergency Medicine Residency Program; Director, Pediatric Education/Quality Improvement and Observation Unit, Cleveland Clinic Foundation,

Cleveland, Ohio

Lynn P. Roppolo, MD, FACEPAssistant Emergency Medicine Residency Director,

Assistant Professor of Emergency Medicine, University of Texas Southwestern Medical Center at Dallas, Dallas, Texas

Robert A. Rosen, MD, FACEPMedical Director, Culpeper Regional Hospital,

Culpeper, Virginia; Associate Professor, Emergency Medicine, University of Virginia School of Medicine,

Charlottesville, Virginia

George Sternbach, MD, FACEPClinical Professor of Surgery

(Emergency Medicine), Stanford University Medical Center, Stanford, California

Editorial StaffMary Anne Mitchell, ELS

Managing Editor

Mike GoodwinCreative Services Manager

Jessica HamiltonEditorial Assistant

Lilly E. FriendCME and Subscriptions Coordinator

Marta FosterDirector and Senior Editor

Educational and Professional Publications

Critical Decisions in Emergency Medicine is a trademark owned and published monthly by the American College of Emergency Physicians, PO Box 619911, Dallas TX 75261-9911. Send address changes to Critical Decisions in Emergency Medicine, PO Box 619911, Dallas TX 75261-9911, or to [email protected] 2011 © by the American College of Emergency Physicians. All rights reserved. No part of this publication may be reproduced, stored, or transmitted in any form or by any means, electronic or mechanical, including storage and retrieval systems, without permission in writing from the Publisher. Printed in the USA.

Magnesium sulfate

Mechanism of action

Alters contractility of skeletal and smooth muscle. Ability to inhibit acetylcholine in motor nerve terminals. Slows sinoatrial node impulse formation and increases conduction time. In the lungs, magnesium acts to promote relaxation of the bronchial smooth muscle. Enzymatic cofactor in many reactions and ion channels/pumps.

Indications Torsade de pointes Eclampsia/severe preeclampsia HypomagnesemiaOff-label: severe refractory asthma

Dosing Adults. Hypomagnesemia: up to 1-2 g/hr for 3-6 hours IV then 0.5-1 g/hr as needed; if symptomatic 1-2 g over 5-60 mins IVTorsade de pointes or polymorphic ventricular tachycardia: 1-2 g IV pushEclampsia/severe preeclampsia: 6 g IV bolus followed by 1-2 g/hr continuous infusionUnlabeled: severe asthma: 2 g IV over 20 minPediatrics. Asthma: 25-75 mg/kg IV (max: 2 g)Hypomagnesemia: 25-50 mg/kg/dose over 10-20 mins (max: 2 g)

Side effects Respiratory depression, hypotension, heart block, weakness with hypotonia and loss of deep tendon reflexes, nausea and vomiting, warmth, flushing, sweating

Contraindications/precautions

Contraindications: heart blockCaution: renal impairment; neuromuscular disease, especially myasthenia gravis; bradycardiaPregnancy category A

Magnesium Sulfate TraceyBanks-Greczanik,DO,AkronGeneralMedicalCenter

Magnesium sulfate is used in the emergency department for many disease processes such as hypomagnesemia, tachyarrhythmias, and seizure abortion in pregnancy. It can also be considered for off-label use for severe asthma. Onset of action for intravenous dosing is immediate; onset for intramuscular dosing can take up to 1 hour. Guidelines suggest giving it no faster than 150 mg/min, and caution is advised when administering IV push as hypotension and asystole can occur. Signs of toxicity include hypotonia, hypotension, and central nervous system depression.

Feature Editors: Michael S. Beeson, MD, MBA, FACEP, and Steven Warrington, MD